CSC - Question Bank - Vol 1

July 6, 2016 | Author: Khushbu Mistry | Category: N/A
Share Embed Donate


Short Description

Canadian Securities Course - Vol 1 - 2015...

Description

CSC VOLUME ONE: Chapters 1 – 3, Test #1 1.

Which of the following statements do you disagree with? a) b) c) d)

2.

Which of the following statements do you agree with in respect to private equity? a) b) c) d)

3.

b)

c) d)

if you want to buy the stock right now you must pay $15.00 and if you want to sell the stock right now you will receive $15.25. if you want to buy the stock right now you should pay between $15.01 and $15.24 and if you want to sell the stock right now you will receive between $15.01 and $15.24. if you want to buy the stock right now you must pay $15.25 and if you want to sell the stock right now you will receive $15.00. market makers are not responsible for posting bid-offers.

A liquid market implies all of the following except… a) b) c) d)

6.

an investor is buying shares from another investor. an investor is buying shares from the issuing company. the issuing company is buying shares from an investor. trading in stocks does not take place on the secondary market.

A market maker has posted a bid-offer of $15.00 – $15.25 for an oil and gas company, ENR Energy. This would tell the market that… a)

5.

It is less risky than investment in public equity. Institutions are forbidden from private equity investing. The asset class, “private equity” includes both debt and equity. Private equity investments tend to be more liquid than public equity.

When stock trading takes place on the secondary market… a) b) c) d)

4.

Capital is mobile and sensitive. When capital is scarce, it becomes more expensive. An example of direct investing is government building a new hospital. Buying stocks is an example of direct investing while buying corporate bonds is an example of indirect investing.

frequent sales. long trading hours. small price fluctuations from sale to sale. narrow price spread between bid and offering price.

On what exchange in Canada are income trusts most likely to trade? a) b) c) d)

TSX Venture. Toronto Stock Exchange. Montreal Exchange ICE Futures Canada.

7.

Almost all bonds and debentures are sold through… a) b) c) d)

8.

The two major types of markets where stocks trade are… a) b) c) d)

9.

bonds. options. derivatives. mutual funds.

The three categories of firms in the Canadian investment marketplace are… a) b) c) d)

12.

SEC. OSFI. IIROC. The trading activity of ATSs is not subject to regulation.

CanDeal was established to facilitate trading in… a) b) c) d)

11.

dealer markets and listed markets. dealer markets and auction markets. listed markets and unlisted markets. unlisted markets and over-the-counter markets.

Which body regulates the trading activity of alternative trading systems (ATSs)? a) b) c) d)

10.

dealer markets. auction markets. Bonds are sold through dealer markets. Debentures are sold through auction markets. Bonds are sold through auction markets. Debentures are sold through dealer markets.

retail, wholesale, and integrated. retail, wholesale, and institutional. retail, institutional, and integrated. institutional, boutiques, and integrated.

The portfolio management team is understood to be part of the… a) b) c) d)

front office. middle office. back office. The portfolio management team is a stand-alone division unto itself.

13.

An investment dealer holds an inventory of bonds that it sells to its clients. In this case, we would say that… a) b) c) d)

14.

Which of the following is not true about Schedule II Banks? a) b) c) d)

15.

pension plan. open-end fund. closed-end fund. investment fund.

CDIC insures eligible deposits up to _______ per deposit in each member institution. a) b) c) d)

17.

The deposits that they hold are eligible for deposit insurance provided by CDIC. They may not engage in all the types of businesses permitted a Schedule I Bank. They are incorporated and operate in Canada as federally regulated foreign bank subsidiaries. They tend to derive their greatest share of revenue from retail banking and electronic financial services.

Another name for a mutual fund is a(n)… a) b) c) d)

16.

the clients are acting as principals and the investment dealer is acting as an agent. the clients are acting as agents and the investment dealer is acting as an agent. the clients are acting as principals and the investment dealer is acting as a principal. Investment dealers never hold securities in inventory to sell to clients.

$ 50,000 $ 60,000 $ 100,000 $1,000,000

In Canada, regulation of the securities industry is a _________ responsibility. a) b) c) d)

municipal. provincial. federal. territorial.

18.

Ms. Xu had $1,250,000 in an Investment Account and $700,000 in her RRSP account. According to CIPF guidelines, she would be covered for… a) b) c) d)

19.

The general principle underlying Canadian Securities legislation is... a) b) c) d)

20.

caveat emptor – Let the buyer beware! full, true, and plain disclosure of all pertinent facts. approval of the investment merits of each class of securities. providing investors the ability to sue if their investment dealer loses them money.

Front running is… a) b) c) d)

21.

$1,000,000 in her Investment Account. $1,000,000 in her Investment Account or $700,000 in her RRSP account. $1,000,000 in her Investment Account and $700,000 in her RRSP account. $1,250,000 in her Investment Account and $700,000 in her RRSP account.

another name for insider trading. short-selling without declaring the short sale. profitable trading based on enhanced analysis. whereby a registrant trades ahead of a known client.

Continuous public disclosure requirements include all of the following provisions except… a)

b) c)

d)

comparative audited financial statements must be filed within 120 days of financial year-end for companies listed on the TSX Venture Exchange. comparative audited financial statement must be filed within 90 days of financial year-end for senior issuers listed on the TSX. comparative unaudited financial statements must be filed within 60 days of the end of each of the first three quarters of the financial year for companies listed on the TSX Venture Exchange. comparative audited financial statements must be filed within 45 days of the end of each of the first three quarters of the financial year for senior issuers listed on the TSX.

22.

A client, Ms. Huang, was contacted by her Investment Advisor on Monday April 10th and convinced to purchase an IPO issued under full prospectus requirements. On Tuesday April 11th, Ms. Huang received the final prospectus in the mail and called her Investment Advisor, thanking him for it. On Thursday morning – without looking at the document – she called back, attempting to cancel the trade. Which of the following best describes Ms. Huang’s options? a)

b) c) d)

23.

What is a “proxy”? a) b) c) d)

24.

An absentee ballot. A power of attorney. A nominee owner form. A beneficial owner form.

XYZ Company currently holds 15% of the voting shares of DEF Company. What additional percent of the voting shares must it look to purchase in order to be bound by takeover bid provisions? a) b) c) d)

25.

She would not be protected under the Right of Withdrawal because it was over 48 hours since she placed the trade to purchase the securities. She would not be protected under the Right of Rescission because she did not read the prospectus. She would be protected under the Right of Withdrawal because it was less than 48 hours since she received the final prospectus. She would be protected under the Right of Rescission as long as she examined the final prospectus and found that there was at least one material misrepresentation.

5% 20% 35% 85%

Mr. Jones is a Director at XYLON Incorporated. He decides to sell 10,000 shares to raise money for the down-payment of a vacation house in the United States. In order to comply with insider reporting, he must… a) b) c) d)

report the change immediately. report the change within three business days. report the change within ten days. Directors of companies are not subject to insider reporting rules.

Chapters 1 – 3, Test #1 – Answers 1.D

1 – 5.

2.C

1 – 10.

3.A

1 – 11/12.

4.C

1 – 12.

5.B

1 – 13.

6.B

1 – 13.

7.A

1 – 15.

8.B

1 – 12 & 1 – 14.

9.C

1 – 16.

10.A

1 – 17.

11.C

2 – 7.

12.A

2 – 8.

13.C

2 – 10.

14.B

2 – 13.

15.B

2 – 16.

16.C

3 – 5.

17.B

3 – 6.

18.C

3 – 10/12.

19.B

3 – 14.

20.D

3 – 19.

21.D

3 – 21.

22.C

3 – 21/22.

23.B

3 – 22.

24.A

3 – 23.

25.C

3 – 25.

CSC VOLUME ONE: Chapters 1 – 3, Test #2 1.

Which of the following best describes the relationship between direct and indirect investment? a)

b)

c)

d) 2.

What occurs when the Government of Canada’s revenues fail to meet expenditures? a) b) c) d)

3.

The Government runs a surplus and it must raise taxes. The Government runs a surplus and it must cut spending. The Government runs a deficit and it must borrow money. The Government runs a deficit and it must spend money.

Private equity’s purchase of debt securities that are trading below par due to financial difficulties at the firm is known as... a) b) c) d)

4.

Direct investment occurs when savers directly buy securities issued by government and corporations, indirect investment occurs when those savers sell the securities to other savers. Indirect investment occurs when the saver buys the securities issued by government and corporations, which in turn use the funds for direct investment in plant, equipment, etc. Direct investment occurs when savers directly buy an ownership position from a corporation, indirect investment occurs when savers purchase debt securities from government or corporations. There is no link between direct and indirect investment.

junk bonds. low-grade debt. distressed debt. bankruptcy debt.

Which of the following is not true with respect to Canadian Exchanges and the products that trade on them? a) b) c) d)

The Toronto Stock Exchange (TSX) lists senior equities and some debt instruments. The Bourse de Montreal trades all financial and equity futures and options. The ICE Futures Canada Exchange (ICE) trades agricultural futures and options. The TSX Venture Exchange trades junior securities, income trusts and ETFs.

5.

The Ontario Securities Commission requires that trades of unlisted securities is reported through the…. a) b) c). d)

6.

Dealer markets are also known as... a) b) c) d)

7.

i) only ii) only i) & ii) only ii & iii) only

Which of the following is not true about CanDeal? a) b) c) d)

9.

bid markets. bid-ask markets. listed markets. unlisted markets.

Which of the following is/are true of trading on the unlisted market? i) A dealer acts as market maker posting bid and offer prices for the securities being traded. ii) The market makers hold an inventory of securities in which they have agreed to “make a market. iii) Brokers do not charge commissions, instead generating revenue from the spread between bid and offer prices. a) b) c) d)

8.

CUB. CBID. QTRS. Trades of unlisted securities do not have to be reported in Canada.

It is operated by the TMX group. It is recognized as both an ATS and investment dealer. It is a joint venture between Canada’s six largest investment dealers. It offers both institutional and retail investors access to Government securities and money market instruments.

Which of the following is not true about CanPX? a) b) c) d)

It operates both retail and institutional marketplaces. It provides investors with real time bid and offer prices. Issues include Government of Canada bonds and treasury bills and provincial bonds. It is a joint venture of the Investment Industry Association of Canada (IIAC)/IIROC member firms.

10.

Which of the following is not a trend that has been recently observed in financial markets? a) b) c) d)

11.

Smaller securities dealers are known as... a) b) c) d)

12.

b) c) d)

They are provincially regulated. They are branches of foreign institutions. They focus on corporate and institutional finance. Their market share in Canada is less than the Schedule I Banks’.

In the context of the insurance industry, the word “underwriting” is most closely associated with... a) b) c) d)

15.

Voting shares must be widely held, restricting control to any one shareholder or group to no less than 20%. Voting shares must be widely held, restricting control to any one shareholder or group to no more than 20%. Voting control must be restricted, with one Canadian-based shareholder or group holding at least 20% of the voting shares. Voting control must be restricted, with one Canadian-based shareholder or group holding at least 50% of the voting shares.

Which of the following is least likely to true about Schedule III banks? a) b) c) d)

14.

boutique firms. specialist firms. broker-dealer firms. broker-dealer agencies.

Which of the following best describes what a Schedule I Bank is required to do in order to remain a Schedule I Bank? a)

13.

The number of exchanges is increasing. Exchanges are merging to meet the challenge of globalization. ATSs are taking market share away from traditional exchanges. Exchanges are forming alliances, partnerships, and electronic links with exchanges in other countries to facilitate global trading.

evaluating risk. whole life insurance. property insurance. property and casualty insurance.

The “prudent portfolio approach” is most closely associated with… a) b) c) d)

credit unions. mutual funds. pension plans. insurance companies.

16.

Which of the following does OSFI regulate? i) Banks ii) Investment dealers iii) Insurance companies iv) Trust and loan companies a) b) c) d)

17.

The thirteen securities regulators of Canada’s province and territories have joined together to form the… a) b) c) d)

18.

against the bankruptcy of member firms. from unprofessional investment advisors. from companies which disappoint investors with their income reports. against losses suffered from premature redemptions of mutual fund units.

Harjinder Singh had $400,000 in cash and securities with Shaky Ground Investment Dealer. When it declares bankruptcy, the market value of the assets it held was $1,000,000,000; however it only had $800,000,000 in assets to pay off investors. Therefore, CIPF would compensate Mr. Singh in the amount of… a) b) c) d)

20.

SEC CSA IIAC OSFI

The function of CIPF is to safeguard investors… a) b) c) d)

19.

i), ii) & iv) only i), ii) & iv) only i), iii) & iv) only i), ii), iii) & iv)

$ 80,000 $100,000 $600,000 Insufficient information.

Which of the following is not true with respect to arbitration in the securities industry? a) b) c) d)

It’s a method of dispute resolution. The arbitrator’s decision is binding. It’s designed for claims of $100,000 or less. Clients who lose in arbitration may then sue the firm.

21.

All of the following are basic methods the securities acts protect the investing public except… a) b) c) d)

22.

Which of the following is not an unethical trading practice? a) b) c) d)

23.

d)

The investor must not have received the prospectus.. There must have been something ambiguous in the prospectus. There must have been an error in the prospectus, no matter how slight it may have been. There must have been an untrue statement or omission of a material fact in the prospectus.

How long does the Board of Directors have to prepare a directors’ circular after a takeover bid is made? a) b) c) d)

25.

Confirming a transaction where no trade has been executed (bucketing). Causing the last sale of the day to be higher than warranted by market conditions (window dressing). Making a practice of taking the opposite side of the market to clients. Trading for a client’s account prior to trading in the Investment Advisor’s own account (front running).

What is necessary in order for an investor to exercise the right of recession? a) b) c)

24.

disclosure of necessary facts. enforcement of laws and policies. registration of dealers and advisors. analysis of publicly traded securities.

One week Fifteen days One months Ninety days

Which of the following best describes the regulations governing insider trading? a) b) c) d)

Insiders are not allowed to trade under any circumstances. Insiders are allowed to trade as long as they are report their trades prior to making any trade. Insiders are allowed to trade as long as they report their trades within ten days of making any trade. Insiders are allowed to trade and do not have to report their trades to anyone.

Chapters 1 – 3, Test #2 – Answers 1.B

1 – 5.

2.C

1 – 8.

3.C

1 – 10.

4.D

1 – 13.

5.A

1 – 16.

6.D

1 – 15.

7.C

1 – 14/16.

8.D

1 – 17.

9.A

1 – 17.

10.A

1 – 17.

11.A

2 – 7.

12.B

2 – 12.

13.A

2 – 13.

14.A

2 – 15.

15.D

2 – 16.

16.C

3 – 5.

17.B

3 – 6.

18.A

3 – 10.

19.A

3 – 11.

20.D

3 – 13.

21.D

3 – 14.

22.D

3 – 19.

23.D

3 – 22.

24.B

3 – 23.

25.C

3 – 25.

CSC VOLUME ONE: Chapters 4 & 5, Test #1 1.

Microeconomics would be least likely to examine… a) b) c) d)

2.

The value of all goods and services produced in a country in a year is known as… a) b) c) d)

3.

expansion peak contraction trough

Housing starts and manufacturers’ new orders are examples of… a) b) c) d)

6.

prices will fall. corporate profits will increase. more goods and services will be sold. there will be a cycle of demand-pull inflation.

“Inflation is stable… Corporate profits are rising and new business start-ups outnumber bankruptcies”… describes which phase of the business cycle? a) b) c) d)

5.

Net National Product Net Domestic Product Gross National Product Gross Domestic Product

Assume that productivity growth exceeds increases in the unit cost of production. Each of the following is likely except… a) b) c) d)

4.

how minimum wage laws affect the supply of labour. if higher taxes on alcohol would impact the tourism industry. the impact of higher interest rates on investment in capital equipment. the relationship between tuition costs and the decision to attend university versus college.

leading indicators. co-incident indicators. lagging indicators. the Composite Index.

All of the following are considered lagging indicators except… a) b) c) d)

inflation. labour costs. unemployment. personal income.

7.

The official Statistics Canada definition of a recession is… a) b) c) d)

8.

Which of the following are the two key indicators that describe the labour market? a) b) c) d)

9.

i) and ii) only i) and iii) only ii) and iii) only i), ii) and iii)

The natural unemployment rate is often viewed as that rate of unemployment that is… a) b) c) d)

11.

Participation rate and unemployment rate Employment rate and unemployment rate Working-age population and participation rate Working-age population and unemployment rate

There are three general types of unemployment: Structural, cyclical and frictional. Which of the following statements are true about them? i) Structural unemployment may result from a conscious choice about wages ii) Cyclical unemployment rises during the expansion phase of the business cycle iii) Frictional unemployment should be understood as a normal part of a healthy economy a) b) c) d)

10.

a 5% drop in economic activity. two consecutive quarters of declining real GDP. two consecutive quarters of declining nominal GDP. None of the above.

impossible to achieve. consistent with stable inflation. consistent with strong economic growth. consistent with zero structural and frictional unemployment.

Which of the following is not true with respect to factors that affect interest rates? a) b) c) d)

Higher inflation results in higher real interest rates. The greater the default risk, the higher the interest rates. An increase in the savings rate will lower interest rates. A large government deficit raises the demand for capital.

12.

Higher interest rates affect the economy in all the following ways except… a) b) c) d)

13.

The CPI stands for the _______________________ and is a measure of the ___________ in Canada. a) b) c) d)

14.

cost-pull inflation. cost-push inflation. output gap inflation. demand-pull inflation.

The Phillips curve says… a) b) c) d)

17.

4.6% 4.8% 5.0% Insufficient information

Inflation that occurs due to shocks from the supply side of the economy is known as… a) b) c) d)

16.

Cost of Products… spending power Canadian Price Index… cost of living Consumer Price Index… cost of living Customer Price Index…. real GDP growth

At the end of 2011, the CPI stood at 110. At the end of 2012, it was 115. Inflation was closest to… a) b) c) d)

15.

mortgage payments will decline. savers enjoy higher interest income. the cost of capital increases for business. consumers are discouraged from buying homes and other durable goods.

lowering inflation means sacrificing actual GDP. lowering inflation means sacrificing potential GDP. lowering inflation means lowering economic growth. lowering inflation means increasing unemployment.

The single most important component of the current account is… a) b) c) d)

strong demand. investment income. merchandise trade. taxes and import duties.

18.

If Canada is running a deficit in its current account, this means… a) b) c) d)

19.

The Canadian and US dollars are trading at par. If inflation in the United States is expected to be higher than in Canada over the next ten years, we would expect that… a) b) c) d)

20.

Keynesian and Monetarist Keynesian and Supply-Side Monetarist and Supply-Side Rational Expectations and Keynesian

All of the following are major functions of the Bank of Canada except… a) b) c) d)

23.

market forces. the Central Bank. the Federal Government. the International Monetary Fund.

Which economic theories tend to favour minimal government intervention in the economy? a) b) c) d)

22.

interest rates will be higher in the US and its currency will appreciate. interest rates will be higher in the US and its currency will depreciate. interest rates will be lower in the US and its currency will appreciate. interest rates will be higher in the US and its currency will depreciate.

In a floating exchange rate system, the value of a currency is determined by… a) b) c) d)

21.

the nation is not benefiting from trade. the nation is running a deficit in its capital account. the nation is running a surplus in its capital account. the balance of payments is in a state of disequilibrium.

to conduct monetary policy. to issue and remove bank notes. to prepare balanced federal budgets. to act as the government’s fiscal agent.

The overnight rate operates with a band of ____ basis points. a) b) c) d)

25 50 100 200

24.

Special PRAs or SPRAs are used by the Bank of Canada… a) b) c) d)

25.

to narrow the limit of the operating band. to relieve undesired upward pressure on overnight financing rates. to relieve undesired downward pressure on overnight financing rates. to influence call loans and the interbank deposit rates through the large value transfer system.

A redeposit means... a) b) c) d)

that the Chartered Banks are redepositing money with the Bank of Canada which puts upward pressure on interest rates. that the Chartered Banks are redepositing money with the Bank of Canada which puts downward pressure on interest rates. that the Bank of Canada is redepositing money with the Chartered Banks which puts downward pressure on interest rates. that the Bank of Canada is redepositing money with the Chartered Banks which puts upward pressure on interest rates.

Chapters 4 & 5, Test #1 – Answers 1.C

4 – 6.

2.D

4 – 9.

3.D

4 – 11/12.

4.A

4 – 14.

5.A

4 – 16/17.

6.D

4 – 18.

7.D

4 – 18.

8.A

4 – 20.

9.B

4 – 22.

10.B

4 – 22.

11.A

4 – 24.

12.A

4 – 24/25.

13.C

4 – 26/27.

14.A

4 – 26/27. (115 – 110)/110 = 4.6%

15.B

4 – 29.

16.D

4 – 29.

17.C

4 – 32.

18.C

4 – 31/32.

19.B

4 – 32/33.

20.A

4 – 35.

21.C

5 – 6.

22.C

5 – 11.

23.B

5 – 13.

24.B

5 – 14.

25.C

5 – 17.

CSC VOLUME ONE: Chapters 4 & 5, Test #2 1.

Market equilibrium is an important concept in the stock market because… a) b) c) d)

2.

Nominal GDP will always exceed Real GDP unless… a) b) c) d)

3.

this tends to have a deflationary effect. this tends to have an inflationary effect. firms enjoy higher profit margins in the short run. consumers enjoy lower prices in the long run.

“Labour and product shortages cause wage increases… interest rates rise and bond prices fall… stock prices fall and stock market activity declines…” This describes which phase of the business cycle? a) b) c) d)

6.

Increases in population. Improvements in technology. Increases in the capital stock. Increases in government spending.

If productivity growth exceeds the increases in the cost of production, all of the following are true except… a) b) c) d)

5.

there is inflation in the system. there is deflation in the system. the constant dollar approach is used. the constant dollar approach is not used.

Which of the following is least likely to have an impact on a country’s GDP? a) b) c) d)

4.

the supply of a given stock is unlimited. the demand for a given stock will not fluctuate in the short run. the demand for a given stock will not fluctuate in the long run. this establishes the price at which a stock will be exchanged at any point in time.

trough recession or contraction recovery peak

Housing starts and stock prices are two important… a) b) c) d)

leading indicators. co-incident indicators. lagging indicators. determinants of interest rates.

7.

A popular definition of a recession is… a) b) c) d)

8.

If unemployment increases by the same amount that employment decreases… a) b) c) d)

9.

the participation rate will increase. the participation rate will decrease. the participation rate will be unaffected. either a) or b) depending on the phase of the business cycle.

Which of the following is/are not true with respect to unemployment? i) Cyclical unemployment tends to rise during the expansion phase of the business cycle. ii) Frictional unemployment is not a normal part of a healthy economy. iii) Structural unemployment typically leads to shorter periods of unemployment than frictional unemployment. iv) The distinction between frictional and structural unemployment is sometimes difficult to determine. a) b) c) d)

10.

two declining quarters of real GDP. two declining quarters of nominal GDP. four declining quarters of real GDP the same as the Statistics Canada definition: The depth, duration and diffusion of the decline in business activity.

i) & ii) i) & iii) i), ii) & iii) i), ii), iii) & iv)

Interest rates are currently being raised in the United States of America. As a result, the Canadian economy should be impacted in all of the following ways except… a) b) c)

d)

demand for interest-rate sensitive products like automobiles should be expected to decrease. Canadian bonds should be expected to decline in value. American consumers will have to pay more to service their household debt, which means a decline in discretionary spending on goods and services made in Canada. the Canadian dollar should be expected to appreciate.

11.

The price of oil has increased significantly in the past twelve months. An economist would conclude that… a) b) c) d)

12.

Inflation is generally bad for the economy. However, which of the following individuals or institutions might actually benefit from a high inflation environment? a) b) c) d)

13.

disinflation. the Phillips curve. cost-push inflation. demand-pull inflation.

Currently, inflation is 5%, unemployment stands at 6% and the sacrifice ratio is 3. If the government would like to lower inflation by 2%, then… a) b) c) d)

15.

A senior citizen on a fixed income. A government that has issued a great deal of long-term, fixed-rate debt. A business that sells a product whose demand is highly pricesensitive. A company which has borrowed a great deal of money on a shortterm basis.

Europe has a stronger labour movement that does North America. As a result, policy makers in that part of the world must be more conscious of… a) b) c) d)

14.

this must be inflationary. this must be inflationary only if the Consumer Price Index (CPI) has increased over this period of time. this could be inflationary or just a one-time jump in price, reflecting the increased scarcity of oil. this could not be inflationary because a barrel of oil is not one of the 600 products from which the CPI is calculated.

unemployment would be expected to increase to 9%. unemployment would be expected to increase to 12%. GDP would have to be reduced by 3%. GDP would have to be reduced by 6%.

The opposite of inflation is… a) b) c) d)

reflation. deflation. disinflation. a positive output gap.

16.

When Canada has a surplus in its Current Account… a) b) c) d)

17.

If the Canadian dollar appreciates in value… a) b) c) d)

18.

Debts give government the incentive to see inflation grow. High deficits may cast doubt on the government’s ability to repay. High deficits are politically unpopular, leading to a change in leaders. Governments must sell securities, increasing supply and lowering their price.

The theory that advocates government intervention as a means of achieving economic stability and full employment in the economy is known as… a) b) c) d)

20.

this is good for Canadian consumers and bad for Canadian exporters. this is bad for Canadian consumers and good for Canadian exporters. this is bad for Canadian exporters and does not affect Canadian consumers. this is good for Canadian consumers and does not affect Canadian exporters.

Which of the following reasons is least likely to account for why countries with large public-sector debts and deficits are less attractive to foreign investors? a) b) c) d)

19.

the Balance of Payments must be temporarily unbalanced. it is buying more goods and services from foreigners than it is selling to them. Canadians are investing more abroad than foreigners are investing in Canada. there must also be a surplus in the Capital Account.

Monetarist theory. Rational expectations. Keynesian economics. Supply-side economics.

You are examining the following information for XYZ Nation for the past fiscal year: Government expenditure on goods and services: $40m Government transfer payments: $10m Government revenue: $35m Based on these entries, the national debt has… a) b) c) d)

fallen by $5m. fallen by $15m. increased by $5m. increased by $15m.

21.

Which of the following is not one of the roles of the Bank of Canada? a) b) c) d)

22.

The Bank Rate is… a)

b)

c)

d)

23.

assist with moral suasion. relieve undesired upward pressure on interest rates. relieve undesired downward pressure on interest rates. keep interest rates outside the Bank of Canada’s operating band.

Drawdowns and redeposits involve… a) b) c) d)

25.

the minimum rate at which the Bank of Canada will lend money to the chartered banks and other members of the Canadian Payments Association. the maximum rate at which the Bank of Canada will lend money to the chartered banks and other members of the Canadian Payments Association. the interest rate set in the overnight market – the marketplace where major Canadian financial institutions lend each other money on a short-term basis. the interest rate set in the money markets – the marketplace where major Canadian financial institutions lend international financial institutions on a short-term basis.

Sale and Repurchase Agreements (SRAs) are used by the Bank of Canada to… a) b) c) d)

24.

Regulating credit and currency. Controlling and protecting the currency. Promoting the financial welfare of the Dominion. Increasing fluctuations in the general level of production.

moving funds owned by the Bank of Canada in and out of the chartered banks. moving bonds owned by the Bank of Canada in and out of the chartered banks. moving funds owned by the Federal Government in and out of the chartered banks. moving funds owned by the Federal Government in and out of the International Monetary Fund.

In the United States, the Federal Reserve Board is raising interest rates. In order for Fiscal Policy to be consistent with this action, the American government should… a) b) c) d)

cut income taxes. spend more on higher education. reduce tariffs on Chinese imports. spend less on the military and Medicare.

Chapters 4 & 5, Test #2 – Answers 1.D

4 – 7/8.

2.B

4 – 11.

3.D

4 – 11/12.

4.B

4 – 12.

5.D

4 – 14.

6.A

4 – 16/17.

7.A

4 – 18.

8.C

4 – 20/22.

9.C

4 – 22.

10.D

4 – 23/24.

11.C

4 – 26.

12.B

4 – 28.

13.C

4 – 29.

14.D

4 – 29.

15.B

4 – 30.

16.C

4 – 31.

17.A

4 – 32/33.

18.C

4 – 34/35.

19.C

5 – 5.

20.C

5 – 7/8.

21.D

5 – 10.

22.A

5 – 13.

23.C

5 – 15.

24.C

5 – 17.

25.D

5 – 17/18.

CSC VOLUME ONE: Chapters 6 – 7, Test #1 1.

If the yield of a bond is greater than its coupon rate... a) b) c) d)

2.

Canada Savings Bonds should be understood as… a) b) c) d)

3.

b) c) d)

that the Government of Canada reserves the right to call in bonds at any time. that the Government of Canada reserves the right to call in bonds at any time at par value plus accrued interest. that the corporate issuer can call its bond based on a yield spread with an equivalent Government of Canada bond. that the corporate issuer can call the bond at any time, as long as Government of Canada bonds with similar maturities are also callable.

Which of the following bonds would most likely provide the investor with the highest yield? a) b) c) d)

5.

liquid, negotiable, and marketable. marketable and liquid, but not negotiable. marketable and negotiable, but not liquid. liquid, but neither negotiable nor marketable.

The Canada Yield Call feature means... a)

4.

it is trading at a discount. it is trading at a premium. the investor is losing money. the investor is making money.

A 10-year straight bond. A 10-year callable bond. A 10-year retractable bond. A 10-year convertible bond.

Currently, there is a 5% convertible debenture that is trading at 95. Each $1,000 face value is convertible into 20 common shares. Since the convertible bonds were issued, interest rates have increased sharply. Therefore, it is most likely that the price of the common shares is closest to... a) b) c) d)

$45.00 $47.50 $50.00 $55.00

6.

All of the following statements are true with respect to treasury bills except… a) b) c) d)

7.

A real return bond issued by the Government of Canada uniquely protects the investor against… a) b) c) d)

8.

lower yield. higher yield. similar yield. Provinces do not issue guaranteed bonds.

A railway company wants to borrow from the capital markets. You would recommend that it considers issuing… a) b) c) d)

10.

the risk of default. the risk of inflation. the risk of deflation. the risks of inflation and default.

The Province of Ontario issues a provincial bond with a ten-year maturity and a guaranteed bond, also with a ten-year maturity. We would expect that the guaranteed bond has a… a) b) c) d)

9.

they pay interest at maturity. the difference between the issue price and par represents the return on investment. they are short-term government obligations offered in denominations of $1,000 to $1,000,000. they are sold every two weeks at auction by the Minister of Finance through the Bank of Canada.

warrants. collateral trust bonds. subordinated debentures. equipment trust certificates.

A strip bond…. a) b) c) d)

is sold at par and matures at par. It makes regular interest payments. is sold at a discount and matures at par. It makes regular interest payments. is sold at a premium and matures at par. It makes regular interest payments. is sold at a discount and matures at par. It does not make regular interest payments.

11.

Which of the following is least likely to be true of GICs? a) b) c) d)

All GICs are RRSP eligible. Many GICs offer compound interest. GICs can be used as collateral for loans. CDIC does not cover GICs with maturities in excess of five years.

Please refer to the following bond quote for Questions #12 and #13… Issue

Coupon

PQR Int’l

6.00

12.

Yield

101.40

101.60

5.63%

$100,000 $100,000 plus accrued interest $101,400 plus accrued interest $101,600 plus accrued interest

PQR’s credit quality has improved. The company’s business has deteriorated. Moody’s has changed PQR’s debt rating from AA to A. None of the above provides a reasonable explanation.

A bond’s present value is found by… a) b) c) d)

15.

Ask

Since this security was issued, interest rates have increased slightly. Given this fact, which of the following is most likely true? a) b) c) d)

14.

Bid

How much would it cost to buy $100,000 face of this bond? a) b) c) d)

13.

Maturity Date 1 July/10

adding the present value of the bond’s coupon payments and the present value of the bond’s principal to be received at maturity. adding the present value of the bond’s coupon payments and the future value of the bond’s principal to be received at maturity. adding the future value of the bond’s coupon payments and the present value of the bond’s principal to be received at maturity. adding the future value of the bond’s coupon payments and the future value of the bond’s principle at maturity.

What is the present value of the second coupon of a 5% semi-annual pay bond trading at 92, assuming a discount rate of 7%. (Base your answer on $1,000 face.) a) b) c) d)

$21.47 $21.84 $23.34 $43.67

16.

An 8% annual pay bond with face value of $100,000 is currently trading at 92. It has three years to maturity. Should an investor with a 10% discount rate purchase this security? a) b) c) d)

17.

A T-Bill with 80 days to maturity is priced at 99.25. What is its effective yield? a) b) c) d)

18.

2.50% 5.00% 5.32% Insufficient information.

An investor goes into a bank and sees that a one-year GIC is paying 2%. This refers to its… a) b) c) d)

20.

3.40% 3.42% 3.45% 4.32%

A semi-annual bond with a 5% coupon is currently trading at 94. It has four years to maturity. Its current yield is… a) b) c) d)

19.

Yes, because its present value of $95,026 is more than its current price of $92,000. No, because its present value of $95,026 is less than its issue price of $100,000. Yes, because its current market price of $92,000 is less than its issue price of $100,000. No, because the discount rate of 10% exceeds the bond’s 8% coupon.

real return. nominal return. inflation-adjusted return. Both a) & c)

The graph depicting the term structure of interest rates is known as the… a) b) c) d)

yield curve. bond curve. real return graph. nominal return graph.

21.

According to Expectations Theory, if the yield curve is upward-sloping… a) b) c) d)

22.

Duration measures… a) b) c) d)

23.

5-year GOC bond with a 5% coupon. 5-year GOC bond with a 10% coupon. 10-year GOC bond with a 5% coupon. 10-year GOC bond with a 10% coupon.

An investor held a $250,000 face value 5% Government of Canada semi-annual pay bond that matures October 15th, 2020. When it was trading at 98, he sold it. If the trade settles on June 24th, the buyer of the bond owes the seller of the bond accrued interest of… a) b) c) d)

25.

interest rate risk. reinvestment risk. Both interest rate risk and reinvestment risk. Neither interest rate risk nor reinvestment risk.

Which of the following bonds would likely be least volatile? a) b) c) d)

24.

the market expects the economy to slip into recession. the market expects interest rates to be lower in the future. the market expects interest rates to be higher in the future. the market expects more supply to come to the market in the future.

$2,397.26 $3,676.37 $3,869.86 $8,630.14

Bond indexes are least likely to be used… a) b) c) d)

to construct bond index funds. as a performance measurement tool. as a guide to the performance of the bond market. by the Bank of Canada to help conduct monetary policy.

Chapters 6 – 7, Test #1 – Answers 1.A

6 – 9.

2.D

6 – 9/10.

3.C

6 – 11.

4.B

6 – 10/16.

5.B

6 – 13/15.

6.A

6 – 18.

7.B

6 – 19.

8.C

6 – 20.

9.D

6 – 22.

10.D

6 – 10.

11.A

6 – 25.

12.D

6 – 26.

13.A

6 – 26/28.

14.A

7 – 5.

15.C

7 – 5/11. $25/1.0352 = $23.34

16.A

7 – 5/11. Solve for PV: 3 = N… 10 = I/Y... 8 = PMT… 100 = FV… [CPT][PV]… 95.026 The bond’s present value of $95,026 exceeds its $92,000 market price.

17.C

7 – 11/12. .75/99.25 x 365/80 x 100 = 3.45%

18.C

7 – 12. 5/94 x 100 = 5.32%.

19.B

7 – 15/16.

20.A

7 – 16/17.

21.C

7 – 17/18.

22.A

7 – 23.

23.B

7 – 19/23.

24.A

7 – 28/30. Days to next coupon payment: 70 250,000 x .05 x 70/365 = $2,397.26

25.D

7 – 30.

CSC VOLUME ONE: Chapters 6 – 7, Test #2 1.

What increases the leverage potential for bonds and debentures, compared to preferred shares? a)

b)

c)

d)

2.

In November 1999, BCE issued a 20 year bond. Which of the following statements is true? a) b) c) d)

3.

Today, it should be considered a money market instrument. Today, the bond should be considered a short-term bond. Today, the bond should be understood as a medium-term bond. Today, the bond should be understood as a long-term bond.

Which of the following class or classes of bonds are issued with a feature known as the Canada Yield call? a) b) c) d)

4.

Companies are legally required to make interest payments on bonds and debentures; they are not legally obligation to make dividend payments on preferred shares. Companies are legally required to make dividend payments on preferred shares; they are not legally obligated to make interest payments on bonds and debentures. Interest payments on bonds and debentures are not a tax-deductible expense for the corporation; preferred dividend payments are a taxdeductible expense Interest payments on bonds and debentures are a tax-deductible expense for the corporation; preferred dividend payments are not a tax-deductible expense.

Corporate bonds Provincial bonds Government of Canada bonds Government of Canada and Provincial bonds

What is the purpose of the purchase fund provision? a) b) c) d)

It requires companies to retire a set amount of debt every year. It acts as a price support if the price of the bonds falls at or below a stipulated price. It forces investors to surrender some of their bonds if the price ever falls below par value. It forces investors to surrender some of their bonds if the price ever rises above par value.

5.

Which of the following fixed income products would most likely provide an investor the highest return? a) b) c) d)

6.

XYZ Company issued a 10-year convertible bond, each $1,000 face value convertible into 50 common shares. The bond had the forced conversion featured attached to it. That feature would most likely be utilized if the price of the common shares were… a) b) c) d)

7.

debt test. negative pledge. retractable debt test. limitation on sale and leaseback.

Which of the following is not true with respect to the federal government and the Government of Canada bonds that it issues? a) b) b) c)

9.

below $20 per share. at $20 per share. above $20 per share. Insufficient information.

All of the following are common protective provisions found in Canadian corporate bonds except… a) b) c) d)

8.

A 10-year debenture. A 10-year extendible bond. A 10-year retractable bond. A 10-year convertible debenture.

All Government of Canada bonds are callable. Investors assign the highest quality ratings to federal government bonds. The federal government is the largest single issuer of marketable bonds in the Canadian bond market. Canadian bonds will have higher yields than American bonds if the market thinks that Canadian bonds are riskier.

The gains from treasury bills are taxed as… a) b) c) d)

capital gains. interest income. Either capital gains or interest income: It is the investor’s choice. Either capital gains or interest income: It is Revenue Canada’s choice.

10.

Canada Premium Bonds are issued with a higher interest rate than Canada Savings Bonds because… a) b) c)

d)

11.

An investor purchases a real return bond with a real coupon rate of 3% when inflation is 2%. If inflation would increase to 4%, the investor would receive… a) b) c) d)

12.

a lower nominal return and a lower real return. a lower nominal return and a higher real return. a higher nominal return and the same real return. a higher nominal return and a higher real return.

Serial debentures or instalment debentures are typically issued by… a) b) c) d)

13.

Canada Savings Bonds can be held by anyone. Canada Premium Bonds can be purchased by bona fide Canadian residents only. Canada Savings Bonds are available exclusively with the compound interest option. Canada Premium Bonds only provide regular interest. Canada Savings Bonds can be purchased only between October and April of each year. Canada Premium Bonds are only sold one day per year. Canada Savings Bonds can be cashed at their par value plus any accrued interest at any time. Canada Premiums Bonds can be redeemed only once a year without penalty.

corporations. municipalities. provincial governments. the federal government.

You are managing the portfolio of a high net-worth client. There is a strip bond on the market with a very attractive 7% yield. If you had the choice of buying it in your client’s trading account or RRSP account, you would recommend… a) b)

c)

d)

buying it in the trading account because the gains from a strip bond are treated as capital gains. buying it in the trading account because the interest income that your client would receive annually would take care of the tax consequences. buying it in the RRSP account because although interest income is received each year, it is not enough to take care of the tax consequences. buying it in the RRSP account because cash is not received until the strip bond matures, but the client is responsible for making annual cash payments on the interest income from it.

14.

The Bank of Nova Scotia wants to raise investment capital in the United States to finance its expansion. It contracts with a US underwriter to sell a new issue of bonds, denominated in US currency in the US market. This is an example of a(n)… a) b) c) d)

15.

Interest rates are extremely low. A risk-averse investor believes that the stock market is going to do much better than the fixed-income market, but wants the security associated with GICs. You would recommend the purchase of… a) b) c) d)

16.

DBRS Moody’s Canada Standard and Poor’s TSX Bond Rating Service

Which of the following corporate bonds would be most likely to receive an “AAA” rating? a) b) c) d)

18.

laddered GICs. instalment GICs. index-linked GICs. interest-rate linked GICs.

Which of the following is not one of the independent rating services for bonds in Canada? a) b) c) d)

17.

foreign bond. international bond. North American bond. Free Trade Zone bond.

A clothing retailer that recently IPO’d. A forestry company that has a great deal of debt. An insurance company that has been in business for fifty years. None of the above – only government securities receive AAA ratings.

The higher the discount rate… a) b) c) d)

the higher the present value of the bond’s income stream. The present value of the bond’s principal is unaffected. the higher the present value of the bond’s income stream and the higher the present value of the bond’s principal. the lower the present value of the bond’s income stream. The present value of the bond’s principal is unaffected. the lower the present value of the bond’s income stream and the lower the present value of the bond’s principal.

19.

An 7% annual pay bond with face value of $100,000 has two years to maturity and is current trading at 102. Should an investor with a discount rate of 6% purchase this security? a) b) c) d)

20.

A T-Bill with 340 days to maturity is priced at 98.25. What is its effective yield? a) b) c) d)

21.

5.88% 5.92% 5.97% 6.00%

Reinvestment risk is the risk that… a) b) c) d)

23.

1.63% 1.66% 1.88% 1.91%

A semi-annual bond with a 6% coupon is currently trading at 102. It has three years to maturity. Its current yield is… a) b) c) d)

22.

No. The investor will incur a $2,000 loss when the bond matures. No. The bond’s present value of $101,833 is below its $102,000 purchase price. Yes. The bond’s coupon rate exceeds the investor’s discount rate. Yes. The bond’s present value of $107,574 exceeds its $102,000 purchase price.

interest rates may have fallen by the time the bond matures. interest rates may increase and the value of the bond will fall. more debt will be issued and the price of all debt in the marketplace will suffer as a result. coupon payments may have to be reinvested at a lower rate than what prevailed when the bond was issued.

A weakness of Liquidity Preference Theory is that… a) b) c) d)

it does not explain a flat yield curve. it does not explain an upward-sloping yield curve. it does not explain a downward-sloping yield curve. it does not explain the impact of reinvestment risk.

24.

A bond is priced at 102. Its duration is 5. If interest rates increase by 1.5%, the bond will be priced at… a) b) c) d)

25.

94.35 107.50 109.50 109.65

A bond with face value of $250,000 and a 5% coupon matures December 1, 2021. It was sold at a price of 99 on Monday October 29th, 2013. Including accrued interest, approximately how much would the buyer of the bond have to pay the seller? a) b) c) d)

$247,750 $250,000 $252,750 $255,250

Chapters 6 – 7, Test #2 – Answers 1.D

6 – 6.

2.B

6 – 9.

3.A

6 – 11.

4.B

6 – 15/16.

5.A

6 – 12/16.

6.C

6 – 14/15.

7.C

6 – 16.

8.A

6 – 17/18.

9.B

6 – 18.

10.D

6 – 18/19.

11.C

6 – 19.

12.B

6 – 20/21.

13.D

6 – 10.

14.A

6 – 23.

15.C

6 – 25.

16.D

6 – 27.

17.C

6 – 27/28.

18.D

7 – 5/11.

19.B

7 – 5/11. Solve for PV: 2 = N… 6 = I/Y... 7 = PMT… 100 = FV… [CPT][PV]… 101.833 The bond’s present value of $101,833 is below its $102,000 market price.

20.D

7 – 11/12. 1.75/98.25 x 365/340 x 100 = 1.91%

21.A

7 – 12. 6/102 = 5.88%

22.D

7 – 14.

23.C

7 – 18.

24.A

7 – 23/24. 102 x .05 x 1.5 = 7.65 price change. Because interest rates went up, the bond’s price will go down to 94.35.

25.C

7 – 28/30. Days to next coupon payment: 153 250,000 x .05 x 153/365 = $5,239.73 accrued interest 250,000 x .99 = $247,500 principal amount Total cost: $252,739.73

CSC VOLUME ONE: Chapters 8 – 9, Test #1 1.

A group of shares that trade in less than a standard trading unit is called a(n)… a) b) c) d)

2.

Which of the following is least likely to be a benefit of common share ownership? a) b) c) d)

3.

restricted voting. non-voting shares. subordinate voting shares. stock dividend split shares.

If a company orders a 1 for 3 reverse split or consolidation… a) b) c) d)

5.

marketability unlimited liability potential for capital appreciation favourable tax treatment for dividends

All of the following are categories of restricted shares except… a) b) c) d)

4.

odd lot. even lot. uneven lot. broken lot.

there will be fewer shares at a lower price. there will be fewer shares at the same price. there will be fewer shares at a higher price. there will be more shares at a lower price.

An investor purchased 200 shares of ABC Company at $20 per share. When the shares were trading at $50 per share, the company executed a 2 for 1 stock split. A month later when the shares were at $40, the investor sold 200 shares. What would be her profit on those 200 shares? a) b) c) d)

$ 4,000 $ 6,000 $ 8,000 $12,000

Please use the following information for Questions #6 & #7. 52 Weeks High Low $25.00 $16.25 6.

Close $22.90

Change Volume 500,000 –$.25

$22.65 $22.75 $23.15 $23.25

$22.40 $22.90 $23.00 Insufficient information.

0 $.25 $.50 $.75

XYZ common shares are currently $20 per share. They pay an annual dividend of $.40. XYZ Series I Preferred Shares were issued at, and currently trade for $40 with a 5% dividend yield, and are convertible into 1.5 common shares. What is the conversion cost premium? a) b) c) d)

10.

Low $22.40

A company has an issue of cumulative preferred shares outstanding. Its regularly quarterly dividend of $.25 per share has been omitted for two consecutive quarters. Before the common shareholders receive a dividend, the preferred shareholder would have to receive a dividend of _________ per share. a) b) c) d)

9.

High $23.00

What could an investor bought/sold GED at the opening of that day’s trading? a) b) c) d)

8.

Div. .60

What was the previous day’s close for GED? a) b) c) d)

7.

Stock GED

20% 25% 33% 50%

What are the years to re-pay premium? a) b) c) d)

11 years 13 years 15 years 17 years

11.

A 5% $50 par value retractable preferred share is purchased at $49.25 and is retractable at the holder’s option in eighteen months. What is the annual pre-tax yield? a) b) c) d)

12.

One of the least heavily weighted sectors of the S&P/TSX 60 is… a) b) c) d)

13.

It is price-weighted. It is weighted by market capitalization. It is based on a large number of transportation stocks. It draws its members from companies that trade on different exchanges around the world.

An investor believes that the Japanese economy will over-perform the rest of the world over the next decade and wants exposure to its stock market. You would recommend that he invests in the… a) b) c) d)

16.

It consists of 30 stocks. It is market-cap weighted. Its stocks are considered “blue-chip”. Its stocks have a below-average risk profile.

Which of the following is true of the S&P 500? a) b) c) d)

15.

Energy. Materials. Financials. Health Care.

Which of the following is not true with respect to the Dow Jones Industrial Average? a) b) c) d)

14.

5.0% 5.55% 6.05% 11.08%

DAX CAC 40 FTSE 100 Nikkei Stock Average

An investor bought 500 shares of BB on Friday July 15th in a cash account. On what date would she be required to make full payment for this purchase? a) b) c) d)

Friday July 15th Monday July 18th Tuesday July 19th Wednesday July 20th

17.

Interest on a margin loan is… a) b) c) d)

18.

An investor purchased 500 shares of ABC Security at $1.90 per share in early morning trading, promising to deposit sufficient margin before business the following morning. If the shares ranged in price from $1.60 to $2.05 during the day before closing at $1.70, what is the margin he would be required to deposit, assuming that he started the day with no cash or securities in his account? a) b) c) d)

19.

$ 9,900 $16,500 $33,000 $42,900

Refer to #20. GHI subsequently rises to $24. What additional margin must be deposited? a) b) c) d)

22.

$380 $425 $550 $930

An investor short sold 1,500 shares of GHI Security at $22. It was eligible for reduced margin. What is the additional margin deposit he would be required to make? a) b) c) d)

21.

$438 $780 $790 $950

What would the margin surplus be if the next day, the shares closed at $2.20? a) b) c) d)

20.

calculated daily and charged daily. calculated daily and charged weekly. calculated daily and charged monthly. calculated daily and charged annually.

$ 900 $2,700 $3,000 $3,900

Refer to #21. Assume that instead of depositing additional margin, the investor closed out his position. He would lose… a) b) c) d)

$1,500 $3,000 $3,900 $5,000

23.

With respect to short selling, all of the following statements are true except… a) b) c) d)

24.

If an investor wants to sell a security immediately, she should place a… a) b) c) d)

25.

There is a time limit of one year on short sales. The short seller is responsible for maintaining adequate margin in the short account. The short seller is liable for any dividends or other benefits paid during the period the account is short. There can be difficulties in borrowing a sufficient quantity of the security sold short to cover the short sale.

day order. limit order. market order. stop-loss order.

To avoid a build-up of GTC (or open) orders on the firm’s trading books, many firms will limit a GTC order for a specified time period, generally… a) b) c) d)

five days. seven days. thirty days. six months.

Chapters 8 – 9, Test #1 – Answers 1.A

8 – 5.

2.B

8 – 6.

3.D

8 – 9/10.

4.C

8 – 12.

5.B

8 – 12. Adjust the original purchase price by the terms of the split. Instead of buying 200 shares at $20, think of it as buying 400 shares at $10. Therefore, when 200 shares are sold at $40, there is a $30 profit per share which amounts to $6,000.

6.C

8 – 13.

7.D

8 – 13.

8.D

8 – 14/15.

9.C

8 – 19. It costs $40 to buy one preferred share that is convertible into 1.5 common shares. It costs $30 to buy 1.5 common shares. Therefore, it is 33% more expensive: ($40 – $30) / $30.

10.A

8 – 19. The dividend yield of the common shares is 2%: $.40 / $20.00. The preferred shares yield 5%. 33% / (5% – 2%) = 11 years.

11.C

8 – 21/22. The annual dividend is $2.50. The annualized capital gain is $.50. $3 / ($49.25 + $50.00) / 2 = 6.05%.

12.D

8 – 27.

13.B

8 – 28.

14.B

8 – 28.

15.D

8 – 29.

16.D

9 – 5.

17.C

9 – 6.

18.B

9 – 7/9. The cost of the purchase was $950. Loan value is calculated based on the closing day’s price of $1.70: 500 x $1.70 x 20% = $170. Therefore, the investor must deposit margin of $780.

19.A

9 – 7/9. The shares are now worth $1,100. Loan value is 50% of that, or $550. Because the loan value was previously $170, the margin surplus is the difference, or $380.

20.A

9 – 9/12. The original sale amount is 1,500 x $22 = $33,000. Margin required is 30% or $9,900.

21.D

9 – 9/12. The new margin requirement is 130% x 1,500 x $24. The investor received $33,000 for the short sale and deposited an additional $9,900. Therefore, the margin shortfall is $3,900.

22.B

9 – 12. The sell price is $22. The buy price is $24. 1,500 shares traded which results in a $3,000 loss.

23.A

9 – 13.

24.C

9 – 18.

25.C

9 – 19.

CSC VOLUME ONE: Chapters 8 – 9, Test #2 1.

Mr. Singh has bought 500 shares of ABC Company through his investment dealer, TC Securities. The share certificates will be registered in the name of… a) b) c) d)

2.

Which of the following correctly places the dividend chronology in the correct order? a) b) c) d)

3.

i) only ii) only i) & ii) only ii) & iii) only

Subordinate voting shares… a) b) c) d)

5.

pay date, record date, ex-dividend date, and cum-dividend date ex-dividend date, cum-dividend date, record date, and pay date cum-dividend date, ex-dividend date, record date, and pay date record date, ex-dividend date, cum-dividend date, and pay date

In Canada, which of the following receive “favourable” tax treatment? i) interest income ii) dividend income iii) capital gains a) b) c) d)

4.

Mr. Singh only. TC Securities only. Both Mr. Singh and TC Securities. Share certificates are registered in bearer form.

do not carry a right to vote. carry a right to vote only under limited circumstances. carry a right to vote, but another class of shares carries a greater voting right per share. carry a right to vote, subject to a limit or restriction on the number or percentage of shares that can be voted by a person, company, or group.

If a company has 30,000,000 million shares outstanding and is priced at $30 then announces a 3 for 1 split, which of the following would be true? a) b) c) d)

there would be 30,0000,000 shares at a price of $90. there would be 90,000,0000 shares at a price of $30. there would be 10,000,000 shares at a price of $90. there would be 90,000,000 shares at a price of $10.

6.

An investor purchased 500 shares of ABC Company at $12 per share. When the shares were trading at $27 per share, the company executed a 3 for 1 stock split. A month later when the shares were at $8, the investor sold 200 shares. What would be her profit or loss on those 200 shares? a) b) c) d)

$200 loss $200 profit $400 profit $800 profit

Please use the following information for Questions #7 & #8. 52 Weeks High Low $45.00 $18.20 7.

Low $18.20

Close $18.20

Change Volume –$2.25 250,000

The regular quarterly dividend is $.50. The regular annual dividend is $.50. In the past twelve months, RMD has paid $.50 in dividends. The dividend is 50% of what it was in the previous year.

250,000 trades were made the previous day. 250,000 shares were traded the previous day. 125,000 shares were bought and 125,000 were sold. None of the above.

Which of the following investment objectives do Canadian preferred shares meet for Canadian retail investors? a) b) c) d)

10.

High 20.05

What does “Volume 250,000” mean? a) b) c) d)

9.

Div. .50

What does the “Div .50” mean? a) b) c) d)

8.

Stock RMD

safety and income income and liquidity income and tax minimization liquidity and tax minimization

XYZ common shares are currently $10 per share. They do not pay a dividend. XYZ Series I Preferred Shares were issued at, and currently trade for $25 with a 5% dividend yield, and are convertible into 2 common shares. What is the conversion cost premium? a) b) c) d)

5% 20% 25% 40%

11.

What are the years to re-pay premium? a) b) c) d)

12.

The main gauge for measuring the investment performance of institutional investments in the United States is the… a) b) c) d)

13.

They are always segregated. They are not payable upon demand. By law, the firm cannot pay interest on them. They may be used in the conduct of the firm’s business.

What is the document that a potential margin client is required to obtain before opening a margin account? a) b) c) d)

16.

DAX CAC 40 FTSE 100 Nikkei Stock Average

Which of the following is true of free credit balances in clients’ accounts? a) b) c) d)

15.

S&P 500. Dow Jones 30. S&P/TSX Composite. Dow Jones Industrial Average.

An investor wants to have exposure to the French stock market. You would recommend that she invest in the… a) b) c) d)

14.

2.5 years 4 years 5 years 10 years

New Account Application Form Margin Account Agreement Form Collateralized Margin Account Form Noncollateralized Margin Account Form

An investor buys 500 shares of an option-eligible security at $22. What is the margin requirement? a) b) c) d)

$ 3,300 $ 5,500 $ 7,700 $11,000

17.

Refer to Question #16: Assume the security subsequently rises to $25. If when the security was $22 the investor had exactly enough money in his/her account, what would his margin deficit or surplus be after the increase? a) b) c) d)

18.

An investor has $1,200 in her account. She asks her broker to buy 1,200 shares of BCE @$28, an option-eligible security, in her margin account. What would the margin call be in this circumstance? a) b) c) d)

19.

b)

c)

d)

withdraw a maximum of $1,000 from her account; however she would be required to pay interest on that extra $1,000 in addition to the original $24,000 she borrowed. withdraw a maximum of $1,000 from her account; however she would not be required to pay interest on that extra $1,000 in addition to the original $24,000 she borrowed. withdraw a maximum of $1,400 from her account; however she would be required to pay interest on that extra $1,400 in addition to the original $33,600 she borrowed. withdraw a maximum of $1,400 from her account; however she would not be required to pay interest on that extra $1,400 in addition to the original $33,600 she borrowed.

An investor short-sold 1,000 shares of a common stock at $18. What is the total amount of margin that must be maintained in the account? a) b) c) d)

21.

$ 8,880 $10,080 $15,600 $16,800

An investor purchased 1,000 shares of Regal Bank at $48. It is option eligible. She put down the minimum margin required and borrowed the rest. If Regal Bank shares increased to $50, she could… a)

20.

$ 750 deficit $1,050 surplus $1,225 deficit $1,500 surplus

$ 9,000 $18,000 $23,400 $27,000

Referring to Q#20… if he closed the position when the shares were $16, what is the profit or loss? a) b) c) d)

$2,000 profit $2,000 loss $16,000 profit $ 4,000 loss

22.

Traders generally prefer to confine short sales to companies… a) b) c) d)

23.

An investment dealer borrows shares from Smith’s account. The shares are sold, and held by Brown when the company declares a $.50 per share dividend. How much dividend income will Smith and Brown see on a per-share basis? a) b) c) d)

24.

Smith will receive $.50 and Brown will receive nothing. Smith will receive $.25 and Brown will receive $.25. Smith will receive nothing and Brown will receive $.50. Smith will receive $.50 and Brown will receive $.50.

A client enters a limit order for the day, to buy 1,000 shares of ABC Company at $20.00. The lowest price it trades that day is $20.05. Therefore… a) b) c) d)

25.

having a small number of shares outstanding that are widely held. having a large number of shares outstanding that are widely held. having a small number of shares outstanding that are not widely held. having a large number of shares outstanding that are not widely held.

the order automatically expires and the investor is not charged any commission. the order automatically expires and the investor is charged a nominal commission only. the order automatically renews the next day and the investor is not charged any commission. the order automatically renews the next day and the investor is charged a nominal commission only.

The Professional (Pro) order rule is designed to… a) b) c) d)

ensure that pros get filled before clients. ensure that pros get filled at better prices than clients. ensure that clients get filled before pros. ensure that clients get filled at better prices than pros.

Chapters 8 – 9, Test #2 – Answers 1.B

8 – 5.

2.C

8 – 7/8.

3.D

8 – 10/12.

4.C

8 – 10.

5.D

8 – 12.

6.D

8 – 12. Adjust the original purchase price by the terms of the split. Instead of buying 500 shares at $12, think of it as buying 1,500 shares at $4. Therefore, when 200 shares are sold at $8, there is a $4 profit per share which amounts to $800.

7.C

8 – 13.

8.B

8 – 13.

9.C

8 – 16.

10.C

8 – 19. It costs $25 to buy one preferred share that is convertible into 2 common shares. It costs $20 to buy two common shares. Therefore, it is 25% more expensive: ($25 – $20)/$20.

11.C

8 – 19. 25%/(5% – 0%) = 5 years.

12.A

8 – 28.

13.B

8 – 29.

14.D

9 – 6.

15.B

9 – 6.

16.A

9 – 7/9. The total cost of the purchase is $11,000. Because the security is optioneligible, the loan value is 70% and the margin is 30% or $3,300.

17.B

9 – 7/9. The security is now worth $12,500 and the loan value is 70% of that, or $8,750. The loan value was $7,700, therefore the margin surplus is $1,050.

18.A

9 – 7/9. The cost of the purchase is $33,600. Her margin requirement is 30% of that, or $10,080. Because she has $1,200 in the account, the additional margin required is $8,880.

19.C

9 – 7/9. The cost of the purchase is $48,000. She must put down $14,400 in margin, having loan value of $33,600. If the shares go up to $50, the loan value is $35,000, allowing her to withdraw $1,400.

20.D

9 – 10/12. 1,000 x $18 x 150% = $27,000.

21.A

9 – 12/13.. Sell price = $18. Buy price = $16. Profit = $2 x 1,000 shares = $2,000.

22.B

9 – 13.

23.D

9 – 14.

24.A

9 – 19.

25.C

9 – 21.

CSC VOLUME ONE: Chapter 10, Test #1 1.

Which of the following is least likely to be an attractive feature of OTC derivatives for corporations? a) b) c) d)

2.

What is the most important function of the clearing corporation? a) b) c) d)

3.

pays the premium and has the right to buy. pays the premium and has the obligation to sell. receives the premium and has the right to sell. receives the premium and has the obligation to buy.

An investor bought 3 GE OCT 30 Calls @ $2. What does the “30” refer to? a) b) c) d)

6.

reducing risk. increasing risk. reducing risk and increasing return. reducing risk and reducing potential return.

The writer of a put option… a) b) c) d)

5.

To increase liquidity. To guarantee performance. To reduce bid-ask spreads. To standardize contract sizes.

By definition, hedging means… a) b) c) d)

4.

liquidity privacy customization standardization

the strike price the option’s price the potential profit the underlying’s current price

Referring to the Question #5, what would happen in the client’s account immediately upon completion of the purchase? a) b) c) d)

The cash position would be reduced by $600. The cash position would be reduced by $9,000. The cash position would be reduced by $9,600. The cash position would be unaffected until exercise occurred.

7.

European-style options mean that… a) b) c) d)

8.

Which of the following options would be exercised? a) b) c) d)

9.

open orders. open interest. assignment notice. underlying interest.

All of the following are reasons to purchase call options except… a) b) c) d)

12.

in the money. out of the money. having no time value. having no intrinsic value.

The total number of outstanding option contracts for a particular option series is known as… a) b) c) d)

11.

a $20 put purchased for $3 when the underlying is $22 a $40 call purchased for $1 when the underlying is $40 a $50 call purchased for $10 when the underlying is $51 a $30 put purchased for $2 when the underlying is $31

If a put has an exercise price of $45 and the underlying security is $44, we would refer to that option as… a) b) c) d)

10.

the option multiplier is 100. the options are cash settled. they are traded internationally. exercise takes place when the option expires.

to fix a future price. to close out a position. to enjoy the benefits of leverage. to insure against a future drop in price.

If an investor sells a call option but does not own the underlying asset on which the option is based, that investor is known as… a) b) c) d)

naked. covered. risk-averse. openly interested.

13.

A silver mining company knows that it will bring 100,000 ounces of silver to the market in nine months. It believes that the price of silver will be significantly higher at that time, but wants to protect itself against price declines. You would recommend that it… a) b) c) d)

14.

An investor buys a put option with a $25 exercise price. The premium for it is $2. If the option has $2 of time value when it is purchased, what is the break-even price for the investor? a) b) c) d)

15.

both are obligated to participate in the future trade. only the long party is obligated to participate in the future trade. only the short party is obligated to participated in the future trade. neither party is obligated to participate in the future trade.

The two levels of margin used in futures trading are… a) b) c) d)

18.

fall in value. fall in value or stay at the same price. rise in value. rise in value or stay at the same price.

When two counter-parties enter into a forward agreement… a) b) c) d)

17.

$23 $25 $27 $29

A naked put writer who sold at-the-money options would like the underlying security to… a) b) c) d)

16.

buys put options on silver. sells call options on silver. sells silver futures contracts. buys silver forward contracts.

initial margin and minimum margin. original margin and minimum margin. initial margin and maintenance margin. original margin and maintenance margin.

A futures contract is held to expiration. This means that… a) b) c) d)

the long party will have to make payment to the short. the short party will have to make payment to the long. the long party will have to make payment to the short and the short party will have to deliver the underlying asset. the short party will have to make payment to the long and the long party will have to deliver the underlying asset.

19.

Marking-to-market is an important feature of… a) b) c) d)

20.

Companies choose to raise additional funds through a rights offering for all the following reasons except… a) b) c) d)

21.

$ 5,000,000 $ 10,000,000 $ 25,000,000 $100,000,000

A stock trades for $11 per share. It announces a rights offering where 3 rights plus $10 will qualify the investor for one more treasury share. What is the value of one right during the cum-rights period? a) b) c) d)

23.

Current market conditions may not be conducive to an ordinary share issue. The company wants to give existing shareholders the opportunity to acquire additional shares. A rights issue more highly leverages the company, improving ROE. A rights issue allows shareholders to maintain their proportionate interest in the company.

There are 10,000,000 shares outstanding of Company XYZ. It announces a rights offering where each common shareholder receives one right. Ten rights plus $5 allow the investor to purchase one more treasury share. If the rights offering is fully subscribed, how much money will be raised? a) b) c) d)

22.

options trading. futures trading. forward trading. both futures and forward trading.

$ .25 $ .33 $ .50 $1.00

When does the TSX terminate trading in rights? a) b) c) d)

At 10:00 on the expiration day. At 12:00 on the expiration day. At 2:00 on the expiration day. At 4:00 on the expiration day.

24.

A warrant is issued with the following terms: One warrant plus $10 entitles the shareholder to one Treasury shares. The warrants trade for $8. If the shares have a par value of $1 and are trading for $16 at the time of issue, the time value of the warrants would be… a) b) c) d)

25.

$1 $2 $6 $8

What are important differences between rights and warrants? a) b) c) d)

Rights have longer lives and are a means of issuing treasury shares. Rights have shorter lives and reduce the number of outstanding shares. Warrants have longer lives and their issuance dilutes the holdings of current shareholders. Warrants have shorter lives and their issuance dilutes the holdings of current shareholders.

Chapter 10, Test #1 – Answers 1.A

10 – 7/9.

2.B

10 – 8.

3.D

10 – 12.

4.D

10 – 15.

5.A

10 – 20.

6.A

10 – 15.

7.D

10 – 17.

8.C

10 – 18.

9.A

10 – 18.

10.B

10 – 20.

11.D

10 – 21/24.

12.A

10 – 25/26.

13.A

10 – 27/28.

14.A

10 – 26/27.

15.D

10 – 29.

16.A

10 – 31.

17.C

10 – 32.

18.C

10 – 31.

19.B

10 – 32.

20.C

10 – 36/38.

21.A

10 – 38/39. 10,000,000/10 x 5 = $5,000,000.

22.A

10 – 38/39. ($11 – $10)/(3 + 1) = $.25

23.B

10 – 39.

24.B

10 – 39/40. You could buy the shares right now for $16. The warrants would cost you $8, and then it would take another $10 to buy one share. The time value is the difference between the two alternatives, or $2.

25.C

10 – 36/40.

CSC VOLUME ONE: Chapter 10, Test #2 1.

Which of the following is not true with respect to features common to all derivatives? a) b) c) d)

2.

One of the attractive features of OTC derivatives is that… a) b) c) d)

3.

calls and puts stocks and bonds futures and forwards commodities and financials

Hedging is the attempt to eliminate or reduce the risk of either holding an asset for future sale or anticipating a future purchase of an asset. Hedging with derivatives involves… a) b) c) d)

5.

standardization exists. liquidity – offsetting is possible. there is no default or credit risk. contracts can be custom designed.

The two major categories of underlying assets for derivative contracts are… a) b) c) d)

4.

All derivatives have a price. All derivatives have an expiration date. All derivatives can be considered a zero-sum game. All derivatives require a performance bond or good-faith deposit.

taking a position in a derivative with a payoff that is similar to that of the asset to be hedged. not taking a position in a derivative with a payoff that is similar to that of the asset to be hedged. taking a position in a derivative with a payoff that is opposite to that of the asset to be hedged. Derivatives are not used for hedging purposes.

With respect to option contracts, the specified price at which an investor can buy or sell is known as the… a) b) c) d)

premium. exercise price. opening transaction. in-the-money amount.

6.

An investor wrote 100 MSFT January 30 calls. As a result, he will have the… a) b) c) d)

7.

Options that can be exercised at any time up to and including the expiration date are referred to as… a) b) c) d)

8.

An American style put option that is in the money. A European style call option that is deep in the money. An American style put option that out of the money. A European style call option that is out of the money.

The biggest difference between LEAPS and other option contracts is that LEAPS… a) b) c) d)

10.

Asian-style options. American-style options. European-style options. offsetting transaction options.

Which of the following options would most likely be exercised at any given point in time? a) b) c) d)

9.

right to sell 100 shares. obligation to sell 10,000 shares. right to buy 10,000 shares. obligation to buy 10,000 shares

are riskier. are less risky. have a longer time to expiration. are traded OTC rather than through an exchange.

An opening sell transaction in an option results in a… a) b) c) d)

long position. short position. closing position. offsetting position.

11.

An investor purchased a $20 INTC Call, paying a premium of $7. If Intel were currently trading at $25, we would say that the option has ____ or time value. a) b) c) d)

12.

You are looking at an option and its intrinsic value is greater than its time value. You would most likely conclude that it is… a) b) c) d)

13.

$ 700 $1,500 $2,200 $2,500

LUV Airlines trades on the NYSE. It is concerned that the jet fuel it needs may go up significantly in six months, but it doesn’t want to be locked into buying in case the price declines. You would recommend that LUV… a) b) c) d)

16.

profit from an expected decrease in the price of the stock. profit from an expected increase in the price of the stock. profit from the fact that stocks are volatile. profit from the fact that over time options lose their time value.

An investor purchased one $20 ABC Call @ $3.00. When ABC was $30.00, she exercised the option and subsequently sold ABC when it was trading at $45. What was her total profit? a) b) c) d)

15.

in the money with a short time to expiration. in the money with a long time to expiration. out of the money with a short time to expiration. out of the money with a long time to expiration.

The most popular reason to buy call options is to… a) b) c) d)

14.

$ 2 $ 5 $ 7 $25

buys a forward contract on jet fuel. sells a forward contract on jet fuel. buys six month call options on jet fuel. sells six month call options on jet fuel.

By definition, covered call writers… a) b) c) d)

own the underlying stock and sell calls against that position. do not own the underlying stock but wish to purchase it. own the underlying stock and deliver the calls if assigned. do not own the underlying stock and do not wish to own it.

17.

When a forward contract is traded on an exchange, it is known as a(n)… a) b) c) d)

18.

Which of the following is an example of a futures contract that is always cash settled? a) b) c) d)

19.

original. minimum. cash-settled. maintenance.

Which of the following products are issued by companies to raise capital? i) Rights ii) Warrants iii) Options a) b) c) d)

22.

hedging. speculating. risk management. marking-to-market.

The margin level below which an account balance may not fall is known as… a) b) c) d)

21.

oil gold lumber equity index

One of the important features of futures trading is the daily settlement of gains and losses. This is known as… a) b) c) d)

20.

swap. future. option. forward.

i) only i) & ii) only ii) & iii) only i), ii) & iii)

The exercise price of a right is also known as its offering or _____ price. a) b) c) d)

exercise privilege subscription acceptance

23.

XYZ shares are currently trading at $15 per share. The company undertakes a rights offering where each outstanding share receives one right, and five rights plus $12 will allow the investor to purchase one additional treasury share. What is the value of one right during the ex-rights period? a) b) c) d)

24.

Refer to the information in Question #15. If there were 10,000,000 shares issued and outstanding, how many shares would there be if the rights offering were fully subscribed? a) b) c) d)

25.

$ .50 $ .60 $ 3.00 $12.00

11,000,000 12,000,000 15,000,000 60,000,000

The main attraction of warrants is their… a) b) c) d)

liquidity. leverage potential. longer expiration, compared to rights. shorter expiration, compared to rights.

Chapter 10, Test #2 – Answers 1.D

10 – 6/7.

2.D

10 – 7.

3.D

10 – 10.

4.C

10 – 12.

5.B

10 – 16.

6.B

10 – 15.

7.B

10 – 17.

8.A

10 – 17.

9.C

10 – 17.

10.B

10 – 17.

11.A

10 – 18/19.

12.A

10 – 18/19.

13.B

10 – 21.

14.C

10 – 21/23. The option cost $300. Exercising the option cost $2,000. Cost: $2,300. When the security is sold at $4,500, there is a $2,200 profit.

15.C

10 – 23/24.

16.A

10 – 25.

17.B

10 – 31.

18.D

10 – 32.

19.D

10 – 32.

20.D

10 – 32.

21.B

10 – 36.

22.C

10 – 36.

23.B

10 – 38/39. ($15 – $12)/5 = $.60.

24.B

10 – 38/39. There are 10 million shares, therefore 10 million rights. If it takes five rights to buy one additional share, then 2 million new shares will be created, resulting in 12,000,000 altogether.

25.B

10 – 39.

CSC VOLUME ONE: Chapters 11 – 12, Test #1 1.

In a partnership, the general partners… a) b) c) d)

2.

Which of the following is least likely to be understood as a benefit of incorporation? a) b) c) d)

3.

c) d)

an estimate (or approximation) of future results of a company. verbal authorization given by a shareholder to allow an agent to vote his/her shares at a shareholders’ meeting. written authorization given by a shareholder to allow an agent to vote his/her shares at a shareholders’ meeting. another name for the preliminary prospectus.

A voting trust is most closely associated with… a) b) c) d)

5.

limited liability capital withdrawal transfer of ownership continuity of existence

What is a proxy? a) b)

4.

are responsible for the day-to-day operations and are personally liable for debts of the partnership. are responsible for the day-to-day operations and are not personally liable for debts of the partnership. are not responsible for the day-to-day operations and are personally liable for debts of the partnership. are not responsible for the day-to-day operations and are not personally liable for debts of the partnership.

start-up companies. declining companies. bankrupt companies. restructuring companies.

The most important government securities distributors that maintain a certain threshold of activity are known as… a) b) c) d)

major dealers. primary dealers. competitive tender dealers. non-competitive tender dealers.

6.

The most commonplace method Canadian corporations use to raise debt financing is the… a) b) c) d)

7.

A firm has 120 million shares outstanding and they are trading at $15. It is considering issuing another 5 million shares, and if it does, it expects to raise $70 million of investment capital because the extra supply will depress the price in a marketplace that is becoming increasingly nervous about equities. If it is correct that the new market equilibrium will be $14 per share, after the secondary offering its market capitalization is expected to be… a) b) c) d)

8.

prospectus. simplified prospectus. preliminary prospectus. due diligence report.

Which of the following is the least valid reason why an investment dealer would advise a company to raise capital through a debt rather than an equity offering? a) b) c) d)

10.

$70 million $1.4 billion $1.75 billion $1.875 billion

The document that investment dealers prepare when a negotiation for a new issue of securities begins is referred to as the… a) b) c) d)

9.

negotiated offering. competitive tender. non-competitive tender. closed-competitive tender.

Debt issues do not dilute equity ownership. Debt may be the lowest after-tax cost source of financing. Debt does not imply the responsibility to pay interest if the company is losing money in a given year. Debt increases the rate of return earned by the owners of the corporation if the rate of return on the borrowed money exceeds its cost.

A company has undertaken an IPO of its common shares in March of 2009. This is the first time it has offered any securities to the public. A year later, it decides to issue a series of preferred shares. We would understand this as a(n)… a) b) c) d)

initial public offering. example of a secondary offering. example of secondary trading. sale of non-treasury shares.

11.

The waiting period is the time between… a) b) c) d)

12.

In a bought deal, i) the investment dealer acts as principal ii) the issue must be sold as a private placement iii) the underwriter commits to buy a specified number of securities at a set price a) b) c) d)

13.

additional disclosure. additional costs to the company. additional controls on management. established and visible market value.

Trading in XYZ Company’s shares commenced at its usual time of 9:30 a.m. Eastern Standard Time. Around mid-day, it alerts the exchange that it has a major announcement. As a result, the exchange might invoke a… a) b) b) d)

15.

i) only i) or ii) only i) and iii) only ii) and iii) only

All of the following are understood as disadvantages of listing except… a) b) c) d)

14.

the final prospectus and the time when the issue trades on the market. the preliminary prospectus and the time when the issue trades on the market. the issuance of a receipt for the preliminary prospectus and approval of the final prospectus. the beginning of preparation of the preliminary prospectus and the time when the securities administrators approve it for distribution.

halt in trading delay in trading suspension in trading cancellation of the listing

Which of the following captures the relationship on the statement of financial position?: a) b) c) d)

ASSETS + LIABILITIES = SHAREHOLDERS’ EQUITY SHAREHOLDERS’ EQUITY – ASSETS = LIABILITIES ASSETS = LIABILITIES AND SHAREHOLDERS’ EQUITY CURRENT ASSETS = CURRENT LIABILITIES

16.

A company purchased a machine for $100,000,000. It has an estimated salvage value of $10,000,000 and useful life of 5 years. Assuming that the company used double-declining balance depreciation method, what would the machine’s accumulated depreciation be after two years of use? a) b) c) d)

17.

Refer to Question #16. Assume that at the end of the second year, it sold the machine for $40,000,000. This would result in a… a) b) c) d)

18.

$1.6 million; $1.6 million $1.6 million; $3.2 million $1.6 million; $4.0 million $2.0 million; $4.0 million

Which of the following is not true with respect to goodwill? a) b) c) d)

20.

gain of $4,000,000. gain of $12,000,000. gain of $16,000,000. Insufficient information

A capital asset was purchased for $10 million and is estimated to have a useful life of 5 years. Its salvage value is estimated at $2 million. Under the straight line and declining balance methods, what would be written off as depreciation in the first year? (Straight line first) a) b) c) d)

19.

$36,000,000 $40,000,000 $64,000,000 $72,000,000

It is a tangible asset that is tested annually for impairment. Its value on the balance sheet should be viewed with caution. When the carrying amount of goodwill exceeds its fair value, an impairment loss should be recognized. It appears on consolidated balance sheets as the excess of the amount paid for shares over the net asset value.

Which of the methods of inventory valuation will tend to maximize reported profits in an inflationary environment? a) b) c) d)

FIFO Total Cost Average Cost Weighted Average Cost

21.

Company A owns 55% of the common shares of Company B. According to accounting rules… a) b) c)

d)

22.

In evaluating the financial position of a company, which of the following would be of primary interest? a) b) c) d)

23.

cost method equity income minority interest subsidiary interest

You have a company’s cash flow statement in front of you. In order to determine whether it paid out dividends in the previous year, you would examine the… a) b) c) d)

25.

revenue cash flow revenue and earning power earning power and cash flow

Company A owns 5% of the voting shares of Company B. Therefore, it is most likely going to use the _______ of accounting. a) b) c) d)

24.

Company A would be required to consolidate Company B’s financial statements. Company A would not be required to consolidate Company B’s financial statements, but could elect to if it so chose. Company A would not be required to consolidate Company B’s financial statements, but would be required to do so if Company B so chose. Company would not be required to consolidate Company B’s financial statements unless government regulators required it to.

investing activities section. financing activities section. operating activities section. operating and financing activities sections.

Canadian corporate law… a) b) c) d)

suggests that every limited company appoint an auditor to represent the Board of Directors and report to them annually. requires that every limited company appoint an auditor to represent the shareholders and report to them annually. suggests that every limited company appoint an auditor to represent management and report to shareholders annually requires that every limited company appoint an auditor to represent the Board of Directors and report to shareholders annually.

Chapters 11 – 12, Test #1 – Answers 1.A

11 – 6.

2.B

11 – 7.

3.C

11 – 10.

4.D

11 – 11.

5.B

11 – 14.

6.A

11 – 16.

7.C

11 – 17. 125,000,000 x $14 = $1.75 billion

8.D

11 – 18.

9.C

11 – 19.

10.A

11 – 21.

11.C

11 – 22.

12.C

11 – 24.

13.D

11 – 31/32.

14.A

11 – 33.

15.C

12 – 5.

16.C

12 – 7/8. The depreciation rate is 40%: 100/5 x 2 = 40%. Therefore, in the first year $40,000,000 would be depreciated, leaving $60,000,000 to be depreciated in year two at a rate of 40%, resulting in accumulated depreciation of $64,000,000.

17.A

12 – 7/8. If $64,000,000 has been depreciated, the machine has a net carrying value of $36,000,000. If it is sold for $40,000,000 this represents a $4,000,000 gain.

18.C

12 – 6/8. Straight line depreciation is ($10 million – $2 million) / 5 years or $1.6 million per year. The rate for the double-declining balance method is 40%: 100% / 5 x 2 = 40%, and 40% of $10 million is $4 million.

19.A

12 – 8.

20.A

12 – 9/10.

21.A

12 – 11.

22.D

12 – 12.

23.A

12 – 14.

24.D

12 – 18.

25.B

12 – 19.

CSC VOLUME ONE: Chapters 11 – 12, Test #2 1.

Which of the following investors/owners enjoy limited liability? a) b) c) d)

2.

Which of the following is true of a corporation? a) b) c) d)

3.

primary dealers primary and secondary dealers government securities distributors Any investment dealer registered with IIROC

The Government of Canada needs to raise $5 billion in debt financing. It decides to allocate $100 million of that on a non-competitive tender basis. If the highest winning bid (lowest yield was 2.25%) and the lowest winning bid (highest yield was 2.28%) and the average yield was 2.27%), then what yield would noncompetitive tenders accept? a) b) c) d)

6.

a proxy form an information circular a proxy form and an information circular a proxy form, information circular, and audited financial statements

Which organizations can submit bids to the Bank of Canada for Canadian Government bonds? a) b) c) d)

5.

Corporations cannot issue additional debt. Corporations cannot be sued in a court of law. Shareholders have no liability for the debts of a corporation. Property bought by the corporation belong to the shareholders.

Which of the following must accompany the notice of a shareholders’ meeting? a) b) c) d)

4.

Sole proprietors and shareholders in corporations. General partners and shareholders in corporations. Limited partners and shareholders in corporations. General partners, limited partners and shareholders in corporations.

2.25% 2.27% 2.28% Slightly greater than 2.28%

Which security or securities form a company’s capital stock? a) b) c) d)

common shares only preferred shares only common and preferred shares common, preferred, and debt securities

7.

A company is authorized to issue 10 million common shares with a par value of $1 per share. Through its history, it has issued 6 million and 5 million are currently outstanding. If we were to calculate its earnings per share, we would consider the… a) b) c) d)

8.

Refer to Q#7. Assume that the market price of the common shares is $20. We would say that its market capitalization is… a) b) c) d)

9.

primary dealer. primary offering. broker of record. trust deed restrictions.

Why would an investment dealer recommend that a company issues debentures instead of bonds? a) b) c) d)

11.

$10 million $100 million $120 million $200 million

The right of first refusal on new financings planned by a corporation is most closely associated with the term… a) b) c) d)

10.

5 million shares outstanding. 6 million shares issued. 10 million shares authorized. Any of the above is acceptable.

Debentures come with a lower interest rate than bonds. Debentures provide flexibility because there are no specific pledges or liens. Debentures are more secure than bonds and that makes them more marketable. Debentures are generally issued with longer maturities than bonds, putting off the eventual repayment of principal.

The information circular for in-house use only in the marketing of a new issue is known as the… a) b) c) d)

greensheet. preliminary prospectus. red herring prospectus. preliminary or red herring prospectus.

12.

XYZ Company went public with an IPO of 100 million shares @ $20 per share. The offering included an over-allotment option whereby a total of 105 million were actually sold. The dealer is most likely to exercise the over-allotment option under which of the following circumstances? a) b) c) d)

13.

The NEX is a separate board of which exchange? a) b) c) d)

14.

orders may not be executed. only block orders for institutional investors may be executed. orders for the suspended security are accepted in the unlisted market. orders for the suspended security are accepted if the regulators approve them.

A company began the year with $40 million in assets and $25 million in liabilities. At the end of the year, its assets had increased to $42 million and its liabilities had decreased by $4 million. Its shareholders’ equity would stand at… a) b) c) d)

.

suspension. halt in trading. delay in trading. delayed opening.

A company that is TSX-listed faces a suspension while it works with the exchange to comply with its listing agreement. During the suspension period… a) b) c) d)

16.

TSX CNSX NYSE TSX Venture

At 9:15 Eastern, just before market open, the CEO of XYZ Company suddenly resigns and a press release comes from the company, announcing that it now under investigation by the FBI for securities violations. The exchange would be most likely to announce a… a) b) c) d)

15.

XYZ declares bankruptcy and the shares become worthless XYZ common shares fall to $17 XYZ common shares remain at $20 XYZ common shares rise to $24

$15 million $17 million $19 million $21 million

17.

A company purchased a machine for $50,000,000. It has an estimated salvage value of $10,000,000 and useful life of 10 years. Assuming that the company used straight line depreciation, what would the machine’s accumulated depreciation be after two years of use? a) b) c) d)

18.

Refer to Question #17. Assume that at the end of the second year, it sold the machine for $40,000,000. This would result in a(n)… a) b) c) d)

19.

inventories trade payables prepaid expenses cash and equivalents

A company has $10 billion in bonds outstanding and another $2 billion in accounts payables. When calculating ratios involving debt… a) b) c) d)

22.

Profits will be higher. Liabilities will be lower. Cash flow will be higher. Debt outstanding will be lower.

Which of the following is not a current asset commonly found on the financial statements of many publicly traded companies? a) b) c) d)

21.

loss of $10,000,000 loss of $8,000,000 loss of $2,000,000 gain of $2,000,000

Which of the following is the most important impact of capitalizing an expenditure rather than expensing it? a) b) c) d)

20.

$ 4,000,000 $ 5,000,000 $ 8,000,000 $10,000,000

only the bonds should be included. both the bonds and accounts payables should be included. both the bonds and accounts payables should be included if the accounts payables are overdue. Neither the bonds nor accounts payables should be included.

The ownership threshold for share of profits of associates is traditionally… a) b) c) d)

10%. 20%. 25%. 50%.

23.

Actuarial gains and losses on defined benefit plans and gains and losses from currency translation are found in… a) b) c) c)

24.

The top line of the operating section of the cash flow statement is usually... a) b) c) d)

25.

statement of cash flows. notes to financial statements. other comprehensive income. the statement of financial position.

revenue. gross profit. operating profit. profit.

In the fourth section of the auditor’s report, the auditor indicated that the statements were “fair”. This means… a) b) c) d)

the company was profitable in the previous year. the company’s prospects going forward are favourable. the financial statements were presented accurately and fairly. the financial statements were presented in accordance with International Financial Reporting Standards.

CHAPTERS 11 – 12, TEST #2 – ANSWERS 1.C

11 – 6.

2.C

11 – 6.

3.C

11 – 10.

4.C

11 – 14.

5.B

11 – 14/15.

6.C

11 – 17.

7.A

11 – 17.

8.B

11 – 17.

9.C

11 – 19.

10.B

11 – 19.

11.A

11 – 23.

12.D

11 – 27.

13.D

11 – 30.

14.D

11 – 33.

15.C

11 – 33.

16.D

12 – 5.

17.C

12 – 6/7. $40 million must be depreciated. Therefore, the annual depreciation is $4 million. After two years, the accumulated depreciation would be $8 million.

18.C

12 – 6/7. The machine has a net carrying value of $42 million: $50 million – $8 million. If it were sold for $40 million, there would be a $2 million loss.

19.A

12 – 8.

20.B

12 – 9/10.

21.A

12 – 12.

22.B

12 – 14.

23.C

12 – 12/13.

24.D

12 – 17.

25.D

12 – 19.

HALF EXAM #1 1.

Which of the following is/are true of Canada’s Schedule I Banks? i) They are larger than Schedule II Banks ii) They must be Canadian owned iii) Voting shares must be widely held a) b) c) d)

2.

CIPF protects investors against… a) b) c) d)

3.

Toronto Exchange Ice Futures Canada Canadian Venture Exchange Bourse de Montreal

There is an exchange-traded debenture that is very thinly traded. The posted bid-offer is 85 – 92. We would describe this security as… a) b) c) d)

5.

fraudulent investment advisors volatile capital markets insolvency of member firms unfavourable arbitration decisions

The exchange for financial futures and options in Canada is the… a) b) c) d)

4.

i) only i) & ii) only ii) & iii) only i), ii) & iii)

Liquid, but neither negotiable nor marketable Marketable and liquid, but not negotiable Negotiable and marketable, but not liquid Liquid and negotiable, but not marketable

Which of the follow represents direct investment? a) b) c) d)

An investor buys IBM shares for his RRSP account on the New York Stock Exchange A pension fund participates in a private placement of Ford Commercial Paper IBM uses its excess cash to invest in Treasury bills issued by the US Government Ford purchases IBM software to help manage its supply chain management

6.

Mr. Kumar was called by his broker on Tuesday afternoon and at that time, Mr. Kumar placed an order for 10,000 shares that was being distributed under prospectus requirements. Two days later he received the prospectus, and when he reviewed it over the weekend, he changed his mind and called his broker on Monday morning, and left a message that he was no longer interested in purchasing those 10,000 shares. According to the protections under prospectus requirements... a) b) c) d)

7.

Which of the following is least likely to be considered an “insider” of Black Company? a) b) c) d)

8.

it is importing more than it is exporting it is also running a surplus in its capital account it is running a deficit in its capital account its Balance of Payment are temporarily out of balance

To relieve undesired upward pressure on overnight financing rates the Bank of Canada uses… a) b) c) d)

10.

Black Company itself. Mr. Red, the past CEO. Mr. Pink, the current CEO. Ms. Mauve, owner of 15% of its voting shares.

If Canada is running a surplus in its current account,… a) b) c) d)

9.

he would be protected under the Right of Withdrawal. he would not be protected under the Right of Withdrawal because he failed to call back within two days of placing the order. he would not be protected under the Right of Withdrawal because he failed to call back within two days of receiving the prospectus. he would be protected under the Right of Rescission because information in the prospectus must have disturbed him.

SPRAs or Specials SRAs closed market operations LVTSs

All of the following are closely watched leading indicators except… a) b) c) d)

housing starts manufacturers' new orders stock prices retail sales

11.

All of the following are major functions of the Bank of Canada except… a) b) c) d)

12.

In the __________ phase of the business cycle, falling demand and excess capacity curtail the ability of firms to raise prices. Inflation falls and interest rates follow… a) b) c) d)

13.

d)

the capital account foreign investment in Canada Canadian investment in foreign countries merchandise trade in goods and services

If the Bank of Canada is raising interest rates, in order for fiscal policy to be aligned, the Federal Government should be... a) b) c) d)

16.

Cyclical unemployment is highest when the economy is weak. Frictional unemployment is part of a normal, healthy economy. Structural unemployment tends to last longer than frictional unemployment. If the economy is strong, unemployment should stand at 0%.

The most important component of the current account is… a) b) c) d)

15.

expansion recession contraction trough

Which of the following statements about unemployment are you least likely to agree with? a) b) c)

14.

balancing the federal budget issuance and removal of bank notes acting as the Government's fiscal agent conducting monetary policy

increasing spending and increasing taxes increasing spending and reducing taxes reducing spending and increasing taxes reducing spending and reducing taxes

Which of the following is not true of duration? a) b) c) d)

Duration combines the impact of the coupon rate and term to maturity into one calculation. The higher the duration, the more volatile the price of the bond. Duration is measured in years. The bond’s duration does not change over time.

17.

All of the following are categories of restricted or special shares except: a) b) c) d)

18.

An investor received a dividend from a Canadian corporation in the amount of $100. To get the taxable amount... a) b) c) d)

19.

collateral trust bonds equipment trust certificates first mortgage bonds serial debentures

An investor owns a cumulative preferred share that pays a quarterly dividend of $.50 per share. The company is in arrears for two quarters. If the Board of Directors wanted to pay common shareholders a dividend of $.25 per share, the preferred shareholders would receive… a) b) c) d)

22.

is trading at a discount. is trading at a premium. has lost value. will no longer be redeemable at par.

If a holding company wanted to add debt to its Capital structure, it would issue… a) b) c) d)

21.

the $100 would be grossed up by 15%. the $100 would be grossed up by 38%. the $100 would be grossed up by 50%. the $100 would be grossed up by 75%.

A long-term bond was issued in 2005. Since that time, interest rates have declined sharply. We would expect that this bond… a) b) c) d)

20.

non-voting subordinate voting restricted voting non-restricted voting

$0.00 $ .50 $1.00 $1.50

An investor purchased 1,200 shares of SYZ security at a price of $1.80 per share. What is the margin required? a) b) c) d)

$ 432.00 $ 864.00 $1,080.00 $1,296.00

23.

The shares in Question #22 subsequently increase in price to $2.05. What will happen to the client's margin position? a) b) c) d)

24.

A T-bill with 125 days to maturity is priced at 98.75. What is its yield? a) b) c) d)

25.

$ 3,780 $ 6,300 $12,600 $16,380

The investor who short-sold in Question #25 subsequently offset at $21. What is his profit or loss? a) b) c) d)

27.

1.27% 2.54% 3.65% 3.70%

An investor short-sold 700 shares of a security eligible for reduced margin at a price of $18. What is the margin the investor is required to deposit? a) b) c) d)

26.

There will be a margin surplus of $366.00. There will be a margin surplus of $612.00. There will be a margin surplus of $981.00 There will be a margin deficit of $150.00.

$ 300 profit $2,100 loss $ 700 profit $14,700 loss

Generally speaking, an important difference between sinking funds and purchase funds is that… a) b) c) d)

the sinking fund is a convenience to the holder, the purchase fund a convenience to the issuer. the sinking fund is a convenience to the holder, the purchase fund does not affect the holder. the sinking fund is a convenience to the issuer, the purchase fund helps create a minimum amount of liquidity at prices below par. there are no differences between sinking funds and purchase funds.

28.

The three major bond rating agencies, Dominion Bond Ratings Service, Moody’s Canada and Standard and Poor’s all changed their ratings on XYZ’s bonds from BBB to B. Which of the following is most likely to occur as a result? a) b) c) d)

29.

What happens to the price of a T-Bill as it nears maturity? a) b) c) d)

30.

5.32% 5.38% 5.55% 5.62%

Which of the following bonds would tend to be least volatile? a) b) c) d)

33.

limit order all or none order stop-buy order stop-loss order

An investor purchased a 7% coupon bond with two years to maturity at 103. What is the yield to maturity? a) b) c) d)

32.

It decreases. It is unaffected. It increases. Insufficient information.

An investor short-sold an equity and is looking for an order that will protect her against a sudden price increase. The appropriate order type would be a… a) b) c) d)

31.

The price of XYZ’s bonds would decline in value and the price of its common shares would be unaffected. The price of XYZ’s bonds would decline in value and the price of its common shares would decline in value. The price of XYZ’s bonds would decline in value and the price of its common shares would increase in value. The price of XYZ’s bonds would increase in value and the price of its common shares would decline in value.

A five year bond with a 10% coupon A five year bond with a 12% coupon A three year bond with a 6% coupon A three year bond with a 4% coupon

"Sell my 700 shares of ABC at $14.00… I don't care how long it takes." This is an example of a… a) b) c) d)

all or none order open order good till cancelled good through

34.

A multi-national conglomerate holds an inventory of raw materials in its inventory which it sells at regular periods of time. It believes that commodities will appreciate sharply over the next few months, but wants insurance in case there is a pull-back in prices. You would recommend that the company… a) b) c) d)

35.

buys puts sells calls buys forwards sells futures

What is the present value of the second coupon payment of a 6% semi-annual bond assuming a discount rate of 7%? a) b) c) d)

$28.01 $32.14 $32.99 $56.02

Questions #36 – #38 are based on the following stock quote: 52 WEEKS HIGH LOW STOCK DIV HIGH LOW CLOSE CHANGE VOL. 36.00 23.25 BDF .40 30.00 29.15 29.25 -.45 40,000 36.

What is the lowest price that this security has traded in the past 24 hours? a) b) c) d)

37.

$23.25 $28.80 $29.15 $29.70

If an investor holds 1,000 shares of this security and its dividend policy is standard, how much will that investor receive on the next pay date assuming that she continues to hold the shares? a) b) c) d)

38.

$ 40 $100 $400 Insufficient information

What was the previous day's closing price? a) b) c) d)

$28.80 $29.25 $29.70 $30.00

39.

Why would a company order a reverse stock split or stock consolidation? a) b) c) d)

40.

A company declares a dividend payable Tuesday December 30th to shareholders of record as of Wednesday December 10th. What is the last day that an investor could buy the shares in order to receive that dividend? a) b) c) d)

41.

DAX CAC 40 FTSE 100 S & P 500

A corporation is considering issuing preferred shares. Which of the following type of preferred shares will allow it to minimize its dividend payments? a) b) c) d)

44.

compounding bond strip bond unit bond mortgage bond

An investor believes that the British economy will outperform over the next decade and wants to participate in the anticipated profits. You would recommend that she purchase the… a) b) c) d)

43.

Friday December 5 Monday December 8 Wednesday December 10 Monday December 15

A bond that is sold at a deep discount and does not make any interest payments is known as a zero coupon or… a) b) c) d)

42.

It wants to increase its market capitalization. The share price is currently too high for retail investors. It wants to improve its working capital. It wants to raise the market price of its shares to attract institutional investors.

straight preferred callable preferred convertible preferred non-cumulative preferred

Goodwill appears on the Balance Sheets of companies… a) b) c) d)

that are popular with their customers. that have purchased other companies for more than the latter's book value. that have significant deferred charges as an offsetting position. that do not amortize their intangible assets

45.

Another name for the preliminary prospectus is the… a) b) c) d)

46.

Confirming a transaction where none has taken place is known as… a) b) c) d)

47.

delayed opening. halt in trading. temporary delay. suspension in trading.

Recording an expenditure as an asset rather than an expense is… a) b) c) d)

50.

in order of value. in order of importance. in order of liquidity. in alphabetical order.

If there were a major announcement made by a company that occurred in the middle of the day, the exchange might invoke a… a) b) c) d)

49.

bucketing. high saling. a fictitious trade. ethical trading.

Current assets are listed on a balance sheet… a) b) c) d)

48.

red herring prospectus simplified prospectus greensheet due diligence report

illegal. capitalizing. intangtible. a deferred charge.

You are the treasurer of a company and have wide discretion with respect to selecting accounting procedures. If your company is planning a secondary issue of shares in the near future and wants to report as high a profit figure as possible during a time of rising prices, the preferred inventory method would be… a) b) c) d)

FIFO Average Cost Marginal Cost Total Cost

CSC ONE HALF EXAMINATION #1 - ANSWERS 1D

2C

3D

4C

5D

6A

7B

8C

9A

10D

11A

12D

13D

14D

15C

16D

17D

18B

19B

20A

21D

22D

23A

24D

25A

26B

27C

28B

29C

30C

31B

32C

33C

34A

35A

36C

37B

38C

39D

40A

41B

42C

43C

44B

45A

46A

47C

48B

49B

50A

HALF EXAM #2 1.

Mr. And Mrs. Li are married and both have their investment accounts with DT Securities. Mr. Li has $1,250,000 in his RRSP Account and $500,000 in his Canadian trading account and $800,000 in his US trading account. Mrs. Li has $700,000 in her RRSP account and $575,000 in her Canadian Trading Account. If DT Securities were bankrupt, how much protection would the Li family enjoy? a) b) c) d)

2.

A liquid market is characterized by all of the following conditions except… a) b) c) d)

3.

b) c) d)

She would not be able to get out of the deal because more than 48 hours have passed. She would be able to get out of the deal under the Right of Withdrawal. She would not be able to get out of the deal but is protected under the Right of Action for Damages if the shares decline in price. She would be able to get out of the deal under the Right of Rescission.

Dealer markets are also known as... a) b) c) d)

5.

frequent sales wide price spread between bid and offering prices small price fluctuations from sale to sale narrow price spread between bid and offering prices

An investor purchased 500 shares of an IPO that was issued under prospectus requirements. Three days after receiving the final prospectus in the mail, she notices that there is a material misrepresentation in the prospectus. What best describes her recourse? a)

4.

$1,000,000 $2,000,000 $3,275,000 $3,825,000

listed markets. auction markets. over-the-counter markets. under-the-counter markets

IIROC market surveillance activities include all of the following except... a) b) c) d)

monitoring the margin positions in client accounts carrying out trading analysis and compliance with UMIR. ensuring dealer members comply with timely disclosure. real time monitoring of trading activity on the TSX and TSX Venture and other Canadian exchanges.

6.

What is the time frame within which a directors’ circular must be sent to the security holders of the target company? a) b) c) d)

7.

On the TSX Venture, continuous disclosure requirements mean... a)

b)

c)

d)

8.

lower interest rates in Canada and a weaker Canadian dollar. higher interest rates in Canada and a stronger Canadian dollar. lower interest rates in Canada and a stronger Canadian dollar. higher interest rates in Canada and a weaker Canadian dollar.

"Demand has begun to outstrip the capacity of the economy… labour and product shortages cause wage increases… interest rates rise and bond prices fall…" This describes which phase of the business cycle? a) b) c) d)

10.

audited financial statements must be submitted within 60 days of the end of the quarter and audited financial statements must be submitted within 120 days of the financial year-end. unaudited financial statements must be submitted within 60 days of the end of the quarter and audited financial statements must be submitted within 120 days of the financial year-end. audited financial statements must be submitted within 90 days of the end of the quarter and audited financial statements must be submitted within 120 days of the financial year-end. unaudited financial statements must be submitted within 60 days of the end of the quarter and audited financial statements must be submitted within 120 days of the financial year-end.

If Canada has higher inflation than the United States, this should lead to… a) b) c) d)

9.

48 hours one week fifteen days A directors’ circular is not mandatory, it is optional.

expansion peak recession recovery

The value of all goods and services produced within a country in a given year is measured by… a) b) c) d)

GDP. GNP. the business cycle. exogenous growth.

11.

You read in the newspaper that the Canadian economy has had two consecutive quarters of declining real GDP. This would tell you… a) b) c) d)

12.

The economy is in the expansion phase of the business cycle. Prudent fiscal policy would be to… a) b) c) d)

13.

d)

The balance of payments remains balanced. Canada is importing more than it is exporting. Foreigners are investing more in Canada than Canadian are investing abroad. Canada is suffering through recessionary conditions.

Which school of economic thought believes that the soundest government policy is to cut taxes as much as possible? a) b) c) d)

15.

reduce spending and raise taxes. increase spending and raise taxes. lower interest rates by increasing the money supply. raise interest rates by decreasing the money supply.

If Canada has a deficit in its current account, which of the following is least likely to be true? a) b) c)

14.

that according to the "popular definition" of recession, Canada is in a recession. that according to the Statistics Canada definition of recession, Canada is in a recession. that according to any definition, Canada is in a recession. that the recovery phase of the business cycle will commence in just one more quarter.

Keynesian Monetarists Supply-siders Rational expectations

Which of the following is not true of the Bank Rate? a) b) c) d)

It will always be lower than the Prime Rate. It is the lower limit of the operating band for the Overnight Rate. Lowering the Bank Rate has the impact of stimulating the economy. It is the minimum rate at which the Bank of Canada will lend money to the chartered banks and other members of the CPA.

16.

The primary benefit of a zero coupon bond is that… a) b) c) d)

17.

An investor believes that the French economy will outperform over the next decade and wants to participate in the anticipated profits. You would recommend that she purchase the… a) b) c) d)

18.

stop-loss order limit order market order good through order

An investor wrote 100 put contracts. This means that she... a) b) c) d)

20.

DAX CAC 40 FTSE 100 S & P 500

"I want you to sell my shares immediately because I don't want to lose any more money!" This should be understood as an example of a… a) b) c) d)

19.

the holder does not pay tax until maturity. the difference between the issue price and par value is taxed as a capital gain rather than interest income. sinking fund provisions guarantee liquidity in the after-market. the holder knows with certain the compounded rate of return.

has the right to sell 100 shares has the right to sell 10,000 shares has the obligation to buy 100 shares has the obligation to buy 10,000 shares

Municipalities commonly use this security to raise capital. a) b) c) d)

mortgage bonds savings bonds subordinated debentures instalment debentures

Questions #21 & #22 refer to the following bond quote… ISSUE COUPON MATURITY DATE BID OFFER DEF Company 6.25% 1 Oct 11 97.25 97.75 21. How much would it cost to purchase $100,000 face value of this bond? a) b) c) d) 22.

What is the current yield of the bond? a) b) c) d)

23.

10,000,000 at $45 10,000,000 at $15 90,000,000 at $15 150,000,000 at $45

Canada Savings Bonds would be appropriate for which of the following individuals/institutions: a) b) c) d)

25.

6.25% 6.39% 6.43% Insufficient information

A company is authorized to issue 50,000,000 shares. Currently, it has 30,000,000 outstanding at a price of $45 per share. If a 3 for 1 stock split is issued, how many shares will be outstanding and at what price? a) b) c) d)

24.

$97,250 $97,750 $97,750 plus accrued interest $100,000 less accrued interest

A Canadian investor who believes that interest rates may increase in the near future. A foreign national who wants Canadian dollar exposure in a highly liquid asset. A Canadian investor who hopes to enjoy both income and capital growth from his bond portfolio. A foreign institution that enjoys the opportunity to invest in a bond that pays compound interest .

Which of the following is an example of a Foreign bond? a) b) c) d)

TD Bank issues a bond denominated in Canadian dollars in the American market. TD Bank issues a bond denominated in US dollars in the Canadian market. TD Bank issues a bond denominated in US dollars in the Japanese market. TD Bank issues a bond denominated in US dollars in the US market.

26.

A $250,000 6% semi-annual pay bond that matures October 1, 2020 trades at 99. If it were sold by the holder and the trade settled December 10th, what is the amount of accrued interest that the buyer would owe the seller? a) b) c) d)

27.

A company that is in financial distress has $50 par value preferred shares that are currently trading at $40. If the company is forced into liquidation, how much would the preferred shareholder receive before the common shareholders would get anything for their investment? a) b) c) d)

28.

d)

It helps a corporation when it is restructuring. It helps a company through the IPO phase. It is used when a shareholder meeting must be postponed due to extraordinary circumstances. It is used when a public company is taken private.

If both a Professional Order and a client order compete for the same price and the same security, the preferential trading rule states that… a) b) c) d)

31.

the lower the present value of a bond. the higher the present value of a bond. the lower the coupon rate of the bond. the higher the effective yield of the bond.

What is the purpose of a voting trust? a) b) c)

30.

$10.00 $40.00 $50.00 Common shareholders would be paid out before preferred shareholders.

The higher the discount rate… a) b) c) d)

29.

$2,876.71 $3,132.48 $4,561.64 $7,500.00

priority is always given to the order that was placed first. priority is always given to the client order. priority is always given to the order that will generate the most commission. priority is always given to the Professional's order.

Buy-side firms typically divide investment management duties into… a) b) c) d)

portfolio manager and trader. portfolio manager and analyst. portfolio manager and sell side analyst. sell side analyst and inter-dealer broker.

32.

A company is deciding to issue preferred shares or debentures to raise capital. Which of the following would be a valid reason not to issue preferred shares? a) b) c) d)

33.

Higher inflation will… a) b) c) d)

34.

1.5% 2.9% 3.6% 4.8%

Which of the following bonds would tend to be most volatile? a) b) c) d)

37.

1.01% 1.29% 1.35% 1.37%

An investor purchased a 5% coupon bond with two years to maturity at 104. What is the yield to maturity? a) b) c) d)

36.

increase the nominal rate of return required by an investor. increase the real rate of return required by an investor, decrease the nominal rate of return required by an investor not affect an investor's return.

A T-bill with 270 days to maturity is priced at 99.00. What is its yield? a) b) c) d)

35.

Dividends on preferred shares must be paid while paying interest on debentures is up to the discretion of the Company Treasurer. Dividends on preferred shares are paid with after-tax dollars while the interest on debentures is a tax deductible expense. Dividends on preferred shares are generally paid out annually while interest on debentures is due quarterly. Dividends on preferred shares get less favourable tax treatment for investors than does interest received from debentures

A ten year bond with a 6% coupon A ten year bond with a 12% coupon A twenty year bond with a 6% coupon A twenty year bond with a 12% coupon

An investor received a $200 dividend from a taxable Canadian corporation. For her purposes, the taxable amount would be understood as... a) b) c) d)

$ 30 $200 $276 $0. The corporation has already taken care of the tax obligation.

38.

An investor believes that the German economy will outperform over the next decade and wants to participate in the anticipated profits. You would recommend that she purchase the… a) b) c) d)

39.

DAX CAC 40 FTSE 100 S & P 500

Which of the sectors on the S&P/TSX Composite receives the least amount of weight? a) b) c) d)

Health Care and Utilities Financials and Health Care Energy and Information Technology All 10 Sectors are equally weighted

Please use the following information for Questions #40 & #41. 40.

A company with 100 million shares outstanding declares a rights offering where 10 rights plus $10 will entitle the investor to buy one treasury shares. The common shares currently trade at $11 each. What is the value of one right during the ex-rights period, assuming that the common shares increase to $11.50 per share after the rights offering is declared? a) b) c) d)

41.

Assume that the rights offering was fully subscribed. How many common shares would be outstanding? a) b) c) d)

42.

$.09 $.10 $.14 $.15

10 million 100 million 110 million 200 million

Which of the following is not true with respect to dangers associated with short selling? a) b) c) d)

The seller is theoretically exposed to an infinite loss. The short seller is responsible for paying dividends. The short seller may subject to buy-in requirements at any time. The short seller is not allowed to buy back the shares if the market starts appreciating rapidly.

43.

Under cash account rules, equities must be fully paid... a) b) c) d)

44.

A piece of capital equipment was purchased for $250,000 and is expected to have salvage value of $10,000 after its useful life of 8 years is exhausted. What will be the amount of depreciation written off in year three according to the straight line method? a) b) c) d)

45.

solicition period waiting period waiting time grey market

An incorporated company's Common Shares were issued with a Par Value of $1. An investor bought 1,000 shares at the IPO price of $10 per share, then years later purchased another 1,000 shares at $25 per share. If the company declares bankruptcy and there are 1 million shares outstanding and its obligations total $2 billion, what will the investor owe the various debtors? a) b) c) d)

47.

$30,000 $31,250 $90,000 $93,750

The period of time between the issuance of a receipt for a preliminary prospectus and receipt for a final prospectus is known as the… a) b) c) d)

46.

on the trade date. the business day following the trade date. three days after the trade date. three business days after the trade date.

Nothing because of the principle of limited liability. Nothing because the Board of Directors are personally responsible for the shortfall. $1,000 because that represents the Par Value of the Common Shares. $35,000 because that was the original investment.

Which of the following individuals would be entitled to vote at a shareholders' meeting? i) A registered shareholder ii) The company officers iii) A valid proxy holder a) b) c) d)

i) only i) & ii) only i) & iii) only i), ii) & iii)

48.

All of the following are headings on the cash flow statement except: a) b) c) d)

49.

The key principle(s) behind insider trading regulations is/are to ensure that… a) b) c) d)

50.

operating activities expenditure activities financing activities investing activities

insiders file their intent to buy or sell shares in advance of the purchases or sales. insiders pay their fair share of taxes on the gains they make. insiders report regularly and do not make use of undisclosed information. insiders do not trade under any circumstances

A common reason why a company’s shares are suspended is because… a) b) c) d)

there is a pending merger. the company is without assets or bankrupt. the public distribution has dwindled to an acceptably low level. the company’s financial condition does not meet the exchange’s requirements.

CSC ONE HALF EXAMINATION #2 - ANSWERS 1C

2B

3D

4C

5A

6C

7B

8D

9B

10A

11A

12A

13D

14C

15B

16D

17B

18C

19D

20D

21C

22B

23C

24A

25D

26A

27C

28A

29A

30B

31A

32B

33A

34D

35B

36C

37C

38A

39A

40D

41C

42D

43D

44A

45B

46A

47C

48B

49C

50D

FINAL EXAM 1.

Macroeconomics looks to answer all of the following questions except… a) b) c) d)

2.

Which of the following parties can compete for new issues of Canada Government Debt Securities through the competitive tender system? a) b) c) d)

3.

c) d)

They provide shareholders with limited liability. There is a prohibition against members of the public subscribing to their securities. They have restrictions in their charters on the right of shareholders to transfer shares. They have a limitation which means that there may not be less than fifty shareholders.

Canada Savings Bonds are… a) b) c) d)

5.

government securities distributors primary dealers secondary dealers primary and secondary dealers

Which of the following is not true with respect to private corporations? a) b)

4.

Why did total output shrink this quarter? Will an increase in interest rates slow economic growth? Will a cut in the GST increase automobile sales? How can a nation improve its per capita GDP?

marketable and liquid liquid only marketable and negotiable marketable, liquid and negotiable

If you bought a 225 day T-bill at 97.25, its yield would be… a) b) c) d)

1.74% 2.83% 4.46% 4.59%

6.

“Notice is hereby given that distributions were declared to Shareholders of record as of (Thursday) June 15th payable Friday June 30th for Mill’s Common Shares…” In order to receive this dividend, the latest day you could buy Mill’s Common Shares is… a) b) c) d)

7.

As a general rule, fast-growing companies such as those in the technology field, tend to pay out… a) b) c) d)

8.

All derivatives have a price associated with them. All derivatives have an expiration date. All derivatives have a premium attached to them. Derivatives can be considered a zero-sum game.

IBM issued bonds so that it can build a new research center in India. We would say that… a) b) c) d)

10.

a low percentage of their earnings in the form of dividends. a high percentage of their earnings in the form of dividends. a low dollar amount of their earnings in the form of dividends even though this is typically a high percentage amount. a high dollar amount of their earnings in the form of dividends even though this is typically a low percentage amount.

Which of the following is not one of the features common to all derivatives? a) b) c) d)

9.

Monday June 12th Tuesday June 13th Thursday June 15th Tuesday June 27th

issuing bonds is an example of direct investment. building the research center is an example of indirect investment. issuing bonds is an example of indirect investment and building the research center is an example of direct investment. issuing bonds is an example of indirect investment and building the research center is an example of indirect investment.

A labour economist estimates that the natural unemployment rate in the United States is 6%. If unemployment is currently 10%, then all of the following are true except… a) b) c) d)

Cyclical unemployment is 4%. Cyclical unemployment is 6%. Structural and frictional unemployment is 6%. Structural and frictional unemployment exceeds cyclical unemployment.

11.

The Province of Ontario issues a 10 year bond for a Crown Corporation through a fiscal agent. This is an example of a… a) b) c) d)

12.

What must accompany the notice of a shareholder meeting? a) b) c) d)

13.

to construct bond funds as a performance measurement tool for term extension or reduction as a guide to the performance of the overall bond market

An investor receives a $200 dividend from a taxable Canadian corporation. The dividend tax credit associated with that would be… a) b) c) d)

16.

10-year straight bonds 10-year callable bonds 10-year redeemable bonds 10-year bonds with a mandatory sinking fund provision

Bond indexes are used in all of the following ways except… a) b) c) d)

15.

a proxy form an information circular a proxy form and an information circular a proxy form, an information circular and Annual Report

A firm needs money for a 10-year term and wants to lower its interest expense. You would recommend that it issues… a) b) c) d)

14.

guaranteed bond direct bond depository security fiscal agency agreement

$ 30.00 $ 41.46 $ 76.00 $276.00

A key difference between exchange-traded and OTC derivatives is that… a) b) c) d)

OTC derivatives provide greater standardization. Exchange-traded derivatives provide greater privacy. OTC derivatives provide greater liquidity. Exchange-traded derivatives provide lower default risk.

17.

Preferred shares are best at meeting which of the following investment objectives? a) b) c) d)

18.

You are reading the business section of the newspaper and see that although capacity utilization is low, automobile sales and new home sales are increasing. Unemployment remains high which in turn means that wage pressures are restrained. You would conclude that the economy is in the ______ phase of the business cycle. a) b) c) d)

19.

either a preliminary or red herring prospectus. both a preliminary and red herring prospectus. both a preliminary (or red herring prospectus) and a final prospectus. a final prospectus only.

ABC’s Company Charter authorizes it to issue 10 million common shares. It issues 5 million, then the company declares a 2 for 1 stock split. Subsequently, it buys back 1 million shares in the marketplace. Which of the following is true? a) b) c) d)

21.

contraction trough recovery expansion

A private company is in the process of going public. Most provinces would require that it files… a) b) c) d)

20.

growth and liquidity growth and tax minimization income and tax minimization safety and liquidity

The Company is now authorized to issue an additional 4 million shares. The Company is now authorized to issue an additional 5 million shares. The Company is now authorized to issue an additional 10 million shares. The Company is now authorized to issue an additional 11 million shares.

The forced conversion clause is an advantage to the… a) b) c) d)

the company that issues convertible bonds the investor who buys convertible bonds the investment dealer who underwrites convertible bonds All of the above.

22.

A three year, annual-pay bond with a 6% coupon is currently trading at 104.50 An investor with a 4% discount rate… a) b) c) d)

23.

An investor purchased 1,000 shares of ABC Company at $12 per share. When the company was $25, the company issued a five for one split. She subsequently sold 1,000 shares at $10. Her profit on this transaction would be… a) b) c) d)

24.

20% 23% 25% 27%

A copper-mining company is enjoying the historically high prices and believes that copper prices will trend upward – but fearful that they might pull back significantly in six months. In order to protect its downside yet still enjoy the gains in case copper continues to appreciate in price, it should… a) b) c) d)

26.

$ 2,000 $ 7,600 $10,000 $12,000

NET common shares are currently trading at $20 per share. NET convertible preferred shares with a par value of $50, currently trade for $60 in the market and are convertible into 2.4 common shares. The convertible has a dividend yield of 2.0%. As a result, the preferred share’s conversion cost premium is… a) b) c) d)

25.

would buy the bond because the discount rate is less than the percent by which it is above its issue price would buy the bond because its present value of 105.55 is below its current market price of 104.50. would buy the bond because its present value of 105.55 is above its current market price of 104.50 would not buy the bond because its market price of 105.55 is above its maturity value of 100

buy puts sell calls buy futures sell forwards

In Canada, income trusts and ETFs trade on the… a) b) c) d)

Toronto Stock Exchange (TSX) Toronto Venture Exchange Bourse de Montreal ICE Futures Canada Exchange

27.

Which of the following correctly places an important leading, co-incident and lagging indicator in the correct order? a) b) c) d)

28.

All of the following are true of bought deals except… a) b) c)

d)

29.

ASSETS = LIABILITIES + SHAREHOLDERS EQUITY ASSETS – LIABILITIES = SHAREHOLDERS EQUITY LIABILITIES = ASSETS – SHAREHOLDERS EQUITY SHAREHOLDERS EQUITY = LIABILITIES – ASSETS

All of the following are common protective provisions associated with bonds except… a) b) c) d)

31.

under a bought deal, the dealer acts as a principal. in bought deals, an investment dealer negotiates with the issuer directly. once regulatory approval is received, the bought issue is sold either as a private placement or public issue under the short form prospectus. the spread between the dealer’s cost and final selling price is far higher than under typical, full prospectus underwriting arrangements.

Which of the following does not correctly capture the Balance Sheet relationship? a) b) c) d)

30.

unemployment, retail sales, inflation housing starts, retail sales, unemployment inflation, retail sales, private sector spending on plant and equipment housing starts, GDP, manufacturers’ new orders

security. dividend test. negative pledge. retraction funds.

Which level(s) of government in Canada commonly use installment or serial debentures? a) b) c) d)

municipal provincial federal All of the above

32.

A reverse stock split is utilized when… a) b) c)

d)

33.

An investor had $5,000 in her account at the start of the day. When the market on a security was $1.90 - $1.95, she put in an order to purchase 5,000 shares at the market. She was filled at the ask price and deposited sufficient margin that evening (the security was valued at its bid of $1.90). The next day, the stock closed at $2.20. As a result, she would have a margin surplus of… a) b) c) d)

34.

obligation to buy 100 shares right to buy 100 shares obligation to sell 10,000 shares obligation to buy 10,000 shares

An acronym for the privately-owned computer networks that match orders for securities outside of recognized exchange facilities is… a) b) c) d)

36.

$ 600 $1,600 $1,700 $1,800

An investor wrote 100 put contracts. As a result, he has the… a) b) c) d)

35.

a company wants to reduce the cost of its shares to make them more attractive to retail investors. a company wants to reduce the cost of its shares to make them more difficult to margin. a company wants to increase the cost of its shares when it is in danger of being delisted by a stock exchange because its share price has fallen below the exchange’s minimum share price rule. a company wants to increase the number of its shares to make them more attractive to institutional investors like mutual funds and pension plans.

ATS CUB QTRS IIROC

Which of the following parties might benefit from higher interest rates? a) b) c) d)

A company that has borrowed money in the floating rate market. A young married couple that is looking to purchase their first home. A retiree with a pension that is not indexed to the CPI. A government that has issued a great deal of long-term, fixed coupon debt.

37.

All of the following are advantages of listing securities on a public stock exchange except… a) b) c) d)

38.

A company purchased an asset for $10,000,000 with an estimated useful life of ten years and salvage value of $1,000,000. If the company used the double-declining balance depreciation method, what would the asset’s carrying value be after its second year of use? a) b) c) d)

39.

5 year GOC bond with a 5% coupon 10 year GOC bond with a 5% coupon 5 year GOC bond with a 10% coupon 10 year GOC bond with a 10% coupon

Roger Dodger, investment advisor for FLI-BI-NITE Securities, gets a call from an angry client: “I’m sick of losing money on GOFER BROKE, Inc. I want to stop my losses right now… or at least limit my losses. Sell immediately!” Roger Dodger would place a(n)… a) b) c) d)

42.

yield curve bond market line duration line term extension/reduction curve

Which of the following bonds would be most volatile? a) b) c) d)

41.

$4,600,000 $5,400,000 $5,760,000 $6,400,000

The graph depicting the term structure of interest rates is known as the… a) b) c) d)

40.

prestige and goodwill additional disclosure requirements market visibility facilitates valuation for tax purposes

at the market order limit order stop-loss order all or none order

Which of the following is true with respect to margin accounts? a) b) c) d)

Every investment firm allows margin accounts. Obtaining a Margin Account Agreement Form is optional for the member firm. Interest on the margin loan is calculated daily and charged monthly. Margin refers to the amount the member firm provides.

43.

When a call writer owns the underlying stock, they are known as… a) b) c) d)

44.

Which of the following statements about bond market trading are you least likely to agree with? a) b) c) d)

45.

when inflation is low, unemployment is low when inflation is low, unemployment is high when inflation increases, unemployment increases Inflation and unemployment are not linked.

What is the first step in complying with the “Know Your Client” rule? a) b) c) d)

48.

2.03% 2.07% 2.60% 3.60%

The Phillips curve says that… a) b) c) d)

47.

The sell side of fixed-income trading is the investment dealer side. The buy side of fixed-income trading is the investment management side. Inter-dealer brokers are participants in the retail bond market who bring retail investors together with institutional sellers. The trade ticket, an electronic confirmation, contains information such as the identification of the bond, the price and often the yield, and the settlement date.

At the end of 2013, the CPI was 125.4. At the end of 2014, it was 128. Therefore, the inflation rate for that year was… a) b) c) d)

46.

covered naked cash-secured hedgers

having a lengthy interview with the client before any trade is accepted. doing a background check on the client, verifying employment information and address. completing a New Account Application Form. learning the essential facts relative to every client and every order.

What are two of the most common types of capitalized items? a) b) c) d)

leases and interest plant and equipment amortization and depletion goodwill and deferred charges

49.

All of the following are true of Canada Savings Bonds except… a) b) c) d)

50.

The Spice Girls decide that they want to invest some of their money in the securities markets. Scary Spice buys preferred shares of GIRLPOWER Inc. Sporty Spice buys some first mortgage bonds issued by GIRLPOWER Inc. Baby Spice buys some GIRLPOWER Inc. common shares, and last but not least, Posh Spice buys some GIRLPOWER Inc. convertible debentures. If GIRLPOWER ever faced dissolution, in what order would the Spice Girls get paid off? a) b) c) d)

51.

they are issued with a three year term.. they can be redeemed at any time throughout the year. they are available through the Payroll Savings Program. they can only be transferred to family members.

Sporty, Scary, Posh, Baby Sporty, Posh, Scary, Baby Baby, Scary, Posh, Sporty Sporty, Baby, Posh, Scary

A North American investor believed that political reforms in Japan will mean that its economy would outperform over the next decade. You would recommend that she invest in the… a) b) c) d)

FTSE 100 CAC 40 DAX Nikkei

Please use the following information for Questions #52 & #53. A company has 30,000,000 authorized shares and 20,000,000 that are issued and outstanding. When the common shares (with a par value of $1.00) are trading for $10 in the marketplace, it announces a rights offering where each shareholder will receive one right for each share held, and four rights plus $8.00 will entitle the shareholder to purchase one treasury share. 52.

What is the value of one right during the cum-rights period? a) b) c) d)

53.

$.40 $.50 $2.00 $2.25

If the rights offering is fully subscribed, how many shares will be issued and outstanding? a) b) c) d)

20,000,000 24,000,000 25,000,000 36,000,000

54.

Mr. Singh has numerous accounts with TD Waterhouse which is a member of the IDA and whose accounts are CIPF protected. His Canadian margin account has a market value of $750,000. His American option account has $400,000. His RRSP account is worth $1,100,000 and his in-trust account for his two children has $250,000. How much protection does Mr. Singh have under CIPF rules? a) b) c) d)

55.

$1,000,000 $2,250,000 $2,300,000 $2,500,000

You pull the following information from the Statscan website: Canadian Population: 32,400,000 Working-Age Population: 25,800,000 Labour Force: 17,500,000 Employed: 16,400,000 Unemployed: 1,100,000 What is the participation rate? a) b) c) d)

56.

XYZ Company was supposed to file its audited year-end results with the exchange on Friday June 9th. It failed to do so. As a result, the exchange would likely invoke a _____ on Monday June 12th. a) b) c) d)

57.

delayed opening halt in trading suspension of trading delisting

Deferred revenue results when a company… a) b) c) d)

58.

50.7% 54.0% 63.4% 67.8%

receives payment for goods or services that it has not yet provided. pays for goods or services that it has not yet received. sells a product for less than its cost of goods sold. sells a product for greater than its cost of goods sold.

An investor anticipates that over the years, inflation will erode the real buying power of her returns. To protect herself against this, she would be most likely to purchase what sort of GIC? a) b) c) d)

index-linked GIC instalment GIC laddered GIC escalating-rate GIC

59.

The higher the discount rate… a) b) c) d)

60.

You are sitting on a trading desk when a non-client order and a pro order arrive at your desk. They are competing for the same security at the same price. You would execute… a) b) c) d)

61.

the company’s trading volume. the company’s liquidity. the company’s current profitability. the company’s market capitalization.

A company IPO’d its common shares in May. During the month of June, an investor who participated in the IPO sold her shares to another investor. This is an example of trading on the… a) b) c) d)

63.

the pro order first. the client order first . the order which is for the greatest number of shares. the order which would generate the highest commission.

The people who construct the S&P/TSX Composite are least likely to be interested in… a) b) c) d)

62.

the lower the present value of a bond. the higher the present value of a bond. the lower the future value of a bond. the higher the future value of a bond.

primary market secondary market auction market dealer market

Which of the following type(s) of unemployment is/are tied directly to the fluctuations in the business cycle? a) b) c) d)

cyclical unemployment only cyclical and structural unemployment only cyclical and frictional unemployment only cyclical, structural and frictional unemployment

64.

Ms. Li was called by her broker on Thursday May 11th. She was told about an IPO for TERRUS Common Shares and verbally agreed to buy 1,000 shares. The next morning, her broker mailed out the final prospectus. On Tuesday morning, May 16th, Ms. Li called her broker, asking to cancel her trade. Which of the following best describes her situation? a) b) c)

d)

65.

A company buys back some of its outstanding shares in the course of the year. On the Cash Flow Statement, this transaction would be recorded under… a) b) c) d)

66.

fall in value to 88.32. fall in value to 92.16. increase in value to 99.84. increase in value to 100.

According to Liquidity Preference Theory… a) b) c) d)

68.

Operating Activities Financing Activities Investing Activities This transaction would not be recorded on the Cash Flow Statement.

A bond that was issued at 100 is currently priced at 96. If it has a duration of 8 and interest rates increase by 50 basis points, it will… a) b) c) d)

67.

She would not be allowed to cancel the transaction. She would be allowed to cancel the transaction under the Right of Withdrawal. She would be allowed to cancel the transaction under the Right of Rescission if there were a material misrepresentation in the prospectus. She would not be allowed to cancel the transaction but she would be protected under the Right of Action for Damages.

the yield curve should always tend to be upward sloping. the yield curve should always tend to be downward sloping. the yield curve reflects supply and demand for various maturities. an upward sloping yield curve means that the market expects interest rates to increase in the future.

An investor short-sold 2,000 shares of DEF Security at 21 and offset one month later at 19. If the security was eligible for reduced margin and a dividend of $.25 per share was paid out over this period of time, her return on margin would be closest to… a) b) c) d)

16.7% 19.1% 27.8% 31.8%

69.

Which of the following is not true of the Dow Jones Industrial Average? a) b) c) d)

70.

Which of the following securities is/are issued by companies to raise investment capital? i) Rights ii) Warrants iii) Options a) b) c) d)

71.

i) only i) & ii) only ii) & iii) only i), ii) & iii)

The economy is operating below full capacity. Which economic school/theory believes that it is inherently stable and left alone will quickly adjust to a stable path of growth? a) b) c) d)

73.

i) only ii) only i) & ii) only i), ii) & iii)

Which of the following are self-regulatory organizations (SROs) in the Canadian Financial Services Industry? i) OSFI ii) MFDA iii) IIROC a) b) c) d)

72.

It is price-weighted. It is comprised of 30 blue-chip stocks. It has a lower risk profile than the overall market. It has historically had a higher return than the overall market.

Rational Expectations Theory Keynesian Theory Monetarist Theory Supply-Side Economics Theory

Deliberately causing the last sale for the day in a security to be higher than warranted by the prevailing market conditions is known as… a) b) c) d)

window dressing bucketing front-running showboating

74.

The auditor is responsible for… a) b) c) d)

75.

A Canadian company issued a bond in Germany denominated in US$. This is an example of a… a) b) c) d)

76.

b) c) d)

there can be difficulties in borrowing a sufficient quantity of the security. there is the theoretical possibility of unlimited losses if the shorted security rises dramatically in price. the short seller is liable for any dividends paid while the security is short. short sellers could be responsible for depositing additional margin if the security declines sharply in value.

When the market is at a low… a) b) c) d)

79.

5.0% 5.6% 6.3% Insufficient information.

All of the following are hazards and difficulties of short selling except… a)

78.

foreign bond. foreign-pay bond. Eurobond (Eurodollar). Eurobond (EuroCanadian).

A 5% bond was issued at par and is currently trading at 90. It has ten years to maturity. Its current yield is… a) b) c) d)

77.

preparing the financial statements for a publicly traded company. preparing the financial statements for a publicly traded company and reporting the results to shareholders. representing the Board of Directors and reporting to them annually on the company’s financial statements. representing the shareholders and reporting to them annually on the company’s financial statements.

P/E multiples are low and dividend yields are low. P/E multiples are high and dividend yields are low. P/E multiples are low and dividend yields are high. P/E multiples are high and dividend yields are high.

Which of the following options has the most intrinsic value? a) b) c) d)

A $40 call purchased for $10 when the underlying is $44. A $30 call purchased for $1 when the underlying security is 33. A $50 put purchased for $2 when the underlying is $60. A $100 put purchased for $5 when the underlying is $95.

80.

CIPF protects investors against… a) b) c) d)

81.

Canada currently has a surplus in its Current Account with the United States. As a result, it must have… a) b) c) d)

82.

ask the securities commission permission to sell his shares. warn the securities commission 10 days in advance of his trade. tell the securities commission within 10 days after effecting the trade. Under these circumstances, Robert Gordon would not be allowed to sell his shares.

Generally Accepted Accounting Principles (GAAP) rules… a) b) c) d)

84.

a matching surplus in its Capital Account. a matching deficit in its Capital Account. a matching surplus in its Balance of Payments. a matching deficit in its Balance of Payments.

Robert Gordon is CEO of ABC Company. Knowing that there will be disappointing earnings based on material non-public information, he wants to sell 200,000 of his 300,000 shares. In order to comply with regulations he must… a) b) c) d)

83.

losses due to the financial failure of any firm in the self-regulatory organization. losses due to market volatility. losses due to incompetent financial advisors. losses due to rising interest rates.

provide for flexibility for companies to choose among accounting standards that best reflect their operations. provide for flexibility for companies to choose among accounting standards that allow them to report the highest profit. provide for flexibility for companies to choose among accounting standards that require them to report the lowest profit. do not provide for any flexibility for companies.

All of the following are important bond rating services in Canada except… a) b) c) d)

Dominion Bond Rating Service. Moody’s Canada Inc. Standard & Poor’s Bond Rating Service. Dow Jones Bond Rating Service.

85.

An investor owned a $200,000 face value bond with a 5% coupon, maturing October 20th 2015. He sold the bond to an investor on June 12th at a price of 90, with the trading settling June 15th. What is the accrued interest that the buyer of the bond would owe the seller of the bond? a) b) c) d)

86.

An investor purchases an equity security on Wednesday June 14th. The trade would settle… a) b) c) d)

87.

c) d)

they provide a two-way security. they usually provide a higher yield than the underlying common shares. they usually provide a higher yield than straight preferred shares. they are vulnerable to a decline in price if they are selling off the stock and the price of the common stock declines.

The Bank Rate is the… a) b) c) d)

89.

Wednesday June 14th Thursday June 15th Saturday June 17th Monday June 19th

All of the following are true with respect to convertible preferred shares except… a) b)

88.

$1,306.98 $1,380.96 $1,452.20 $1,534.25

minimum rate at which the Bank of Canada will lend money to the Chartered Banks. maximum rate at which the Bank of Canada will lend money to the Chartered Banks. minimum rate at which the Chartered Banks will lend money to other Chartered Banks. minimum rate at which the Chartered Banks will lend money to their most creditworthy customers.

Which of these bonds is available through the payroll savings plan option? a) b) c) d)

strip bonds convertible bonds instalment bonds Canada Savings Bonds

90.

An investor who short-sold a security and is concerned about limiting her losses in case of an adverse price move in the security would place a… a) b) c) d)

91.

The Bank of Canada uses _____ to offset undesired downward pressure on overnight financing costs. a) b) c) d)

92.

how much GDP must fall to reduce inflation. how much GDP must go up to reduce unemployment. how much inflation must increase to reduce unemployment. how much the exchange rate must fall to increase exports.

If the nominal rate of interest is 6.0% and the inflation rate is 2%, then… a) b) c) d)

95.

short-term to maturity. low duration. high duration. high coupon rates.

The “Sacrifice Ratio” measures… a) b) c) d)

94.

SPRAs SRAs CPAs LVTS

In order to profit from falling interest rates, an investor should seek bonds with… a) b) c) d)

93.

limit order at the market order stop-loss order stop-buy order

the real interest rate is –4.0%. the real interest rate is 3.0%. the real interest rate is 4.0%. the real interest rate is 8.0%.

Which is not one of the three categories of firms in the Canadian financial services system? a) b) c) d)

Retail Integrated Wholesale Institutional

96.

According to Keynesian economic theory, during the contraction phase of the business cycle… a) b) c) d)

97.

In Canada, the regulation of the securities industry is a… a) b) c) d)

98.

b) c) d)

Schedule II banks focus on retail banking and Schedule III banks focus on corporate and institutional finance. Schedule II banks focus on corporate and institutional finance and Schedule III banks focus on retail banking. Both Schedule II and Schedule III banks focus on corporate and institutional business and investment banking. Both Schedule II and Schedule III banks focus on retail banking.

The ownership threshold for a takeover bid in Canada is… a) b) c) d)

100.

provincial responsibility. federal responsibility. shared responsibility between the provincial and federal governments. The securities industry in Canada is not regulated by government.

Generally speaking… a)

99.

government should cut spending and increase taxes. government should increase spending and cut taxes. government should balance budgets. government should eliminate automatic stabilizers.

10%. 20%. 50%. 100%.

Which of the following would tend to lead to an appreciation of the Canadian dollar vis-à-vis the American dollar? a) b) c) d)

Inflation is higher in Canada. Commodity prices are strong. Canada is running a deficit in its current account. Canada is running large federal and provincial budget deficits. .

CSC ONE FINAL EXAMINATION – ANSWERS 1.C

2.A

3.D

4.B

5.D

6.A

7.A

8.C

9.C

10.B

11.A

12.C

13.A

14.C

15.B

16.D

17.C

18.C

19.C

20.D

21.A

22.C

23.B

24.C

25.A

26.A

27.B

28.D

29.D

30.D

31.A

32.C

33.C

34.D

35.A

36.D

37.B

38.D

39.A

40.B

41.A

42.C

43.A

44.C

45.B

46.B

47.C

48.A

49.D

50.B

51.D

52.B

53.C

54.B

55.D

56.C

57.A

58.D

59.A

60.B

61.C

62.B

63.A

64.B

65.B

66.B

67.A

68.C

69.D

70.C

71.C

72.C

73.A

74.D

75.C

76.B

77.D

78.D

79.C

80.A

81.B

82.D

83.A

84.D

85.D

86.D

87.C

88.A

89.D

90.D

91.B

92.C

93.A

94.C

95.C

96.B

97.A

98.A

99.B

100.B

View more...

Comments

Copyright ©2017 KUPDF Inc.
SUPPORT KUPDF